Добавил:
Опубликованный материал нарушает ваши авторские права? Сообщите нам.
Вуз: Предмет: Файл:
3 курс / Патофизиология / База тестов.pdf
Скачиваний:
249
Добавлен:
01.01.2022
Размер:
14.1 Mб
Скачать

ОБЩАЯ НОЗОЛОГИЯ. БОЛЕЗНЬ. ЭТИОЛОГИЯ. ПАТОГЕНЕЗ.

1.У собаки после перерезки седалищного нерва с одной стороны и сделанной буксации по А.Д.Сперанскому на стопе появляется незаживающая рана. Спустя некоторое время такая же язва появляется на симметричном месте другой, здоровой конечности. Каков ведущий механизм возникающих при этом нарушений?

A.Генетический

B.Гуморальный

C.Аллергический

D.Микроциркуляторный

E.*Нейро-гуморальный

2.Больной жалуется на снижение трудоспособности, слабость, озноб. Установлено, что в семье двое заболели гриппом. О каком периоде заболевания можно думать?

A.Латентном

B.*Продромальном

C.Скрытом

D.Разгаре болезни

E.Исходе болезни

3.Инфекционный процесс, вызванный стафилококком, может быть в форме бактерионосительства, ангины, сепсиса, фурункулеза. Чем можно объяснить такую вариабельность клинических проявлений, вызванных одним возбудителем?

A.*Резистентностью организма

B.Патогенностью возбудителя

C.Тропностью возбудителя

D.Путем распространения

E.Путем поступления

4.Ребенок 10-ти лет перенес несколько атак ревматизма. При его клиническом обследовании было установлено, что имели место воспалительные явления в суставах и обнаружились признаки недостаточности митрального клапана. Какое из патологических явлений у данного больного можно отнести к понятию «болезнь»?

A.Артрит

B.Порок митрального клапана

C.*Ревматизм

D.Воспаление суставов

E.Недостаточность митрального клапана

5.В результате перенесенных нескольких атак ревматизма, сопровождавшимися воспалительными явлениями в суставах, у больной сформировалась недостаточность митрального клапана. Какое из патологических явлений у данной больной следует отнести к понятию «патологический процесс»?

A.Ревматизм

B.Порок митрального клапана

C.Недостаточность клапана

D.*Артрит

6.У больного с признаками недостаточности митрального клапана в анамнезе отмечались атаки ревматизма, сопровождавшиеся воспалительными явлениями в суставах. Какое из патологических явлений у данного больного относится к категории «патологического состояния»?

A.*Недостаточность митрального клапана

B.Ревматизм

C.Артрит

D.Воспаление суставов

E.Ревмокардит

7.При рентгенологическом обследовании у больного язвенной болезнью обнаружен стеноз привратника. Это нарушение является:

A.Патологической реакцией

B.Заболеванием

C.Патологическим процессом

D.*Патологическим состоянием

8.После продолжительной и тяжелой болезни у больного снизилось давление (60/40 мм рт.ст.), наблюдается тахикардия, одышка, сознание затемненное. Это состояние можно рассматривать как:

A.Шок

B.Клиническую смерть

C.*Преагонию

D.Агонию

9.У больного, страдающего кариесом, осложненным острым гнойным воспалением околозубной ткани (периодонтитом), сопровождающимся повышением температуры тела, была по показаниям произведена экстракция зуба. Какое из патологических явлений у данного больного следует отнести к понятию «патологическое состояние»?

A.Кариес

B.*Отсутствие данного зуба

C.Периодонтит

D.Воспаление околозубной ткани

E.Лихорадка

10.Мужчина 20-ти лет, принимавший участие в ликвидации последствий Чернобыльской катастрофы, заболел пародонтитом. Какой этиологический фактор наиболее важен в развитии этой патологии?

A.Неполноценное питание

B.*Эмоциональное перенапряжение

C.Повышение физической нагрузки на зубочелюстной аппарат

D.Стрептококки полости рта

E.Дефицит железа

11.У тяжело травмированного человека постепенно наступила биологическая смерть. Свидетельством этого является:

A.Отсутствие подвижности

B.*Аутолиз и разложение в клетках

C.Потеря сознания

D.Отсутствие сердцебиения и дыхания

E.Неупорядоченность химически процессов

12.Мальчик 12-ти лет вернулся из школы и начал жаловаться на головную боль, тошноту, озноб, периодическаю боль в мышцах, потерю аппетита, вялость. Для какого периода болезни характерны такие симптомы?

A.Инкубационного

B.Окончания болезни

C.Латентного

D.Разгара болезни

E.*Продромального

13.У ребенка 5-ти лет, который болеет вирусной инфекцией. Отмечается температура 38,9°С, отсутствие аппетита, боль в мышцах, гиперемия кожи. Какой это период развития болезни?

A.Продромальний

B.Латентный

C.Инкубационный

D.Выздоровления

E.*Разгара болезни

14.У больного туберкулезом легких выделена палочка Коха. До заболевания он много работал, мало отдыхал. Живет в неблагоприятных условиях. Отец раньше тоже болел туберкулезом. Почему именно палочка Коха является причинным фактором туберкулеза?

A.*Абсолютно необходимая и предоставляющая специфические черты.

B.Взаимодействует с другими факторами.

C.Взаимодействует с организмом.

D.Может усугублять течение болезни.

E.Может облегчать течение болезни.

15.У больного гриппом на 5-й день отмечается повышение температуры вечером до 37,1°С, бледность кожных покровов, СОЭ снизилось с 30 мм рт.ст. до 12 мм. рт.ст. Для какого периода болезни характерны такие симптомы?

A.Продромального

B.Латентного

C.Инкубационного

D.Разгара заболевания

E.*Выздоровления

16.У чоловіка, віком 40-ка років, у якого діагностували виразку шлунку, після тривалого стану спокою знов виникли ознаки хвороби. Як треба кваліфікувати такий перебіг хвороби?

A.Ремісія.

B.*Рецидив.

C.Видуження.

D.Латентний період.

E.Продромальний період.

17.У хворого на туберкульоз легень в харкотинні виявлена палочка Коха. Яким фактором є туберкульозна палочка у хворого?

A.Фактор ризику хвороби.

B.Умова розвитку хвороби.

C.*Причинний фактор хвороби.

D.Умова, що перешкоджає розвитку хвороби

E.Умова, що сприяє розвитку хвороби

18.Хворий К., 39-ти років, на протязі останніх 4-х років страждає на виразкову хворобу шлунка. Особливо погіршується його стан восени і навесні: виникають болі у епігастрії, з’являється печія, нудота, закрепи. Яким нозологічним поняттям можна назвати стан хворого в період між загостреннями хвороби?

A.*Патологічний стан

B.Патологічний процес

C.Патологічна реакція

D.Типовий патологічний процес

E.Здоров"я

19.Больной, впервые поступил в стационар с диагнозом - язвенная болезнь желудка. В настоящее время жалуется на боли в эпигастральной области, изжогу, тошноту, дегтеобразный стул. Как можно охарактеризовать такое состояние больного?

A.*Осложнение

B.Рецидив

C.Ремиссия

D.Патологическая реакция

E.Патологическое состояние

20.Відомо, що типові патологічні процеси розвиваються за однаковими закономірностями в різних органах і тканинах та у різних видів тварин. Яке з перерахованих явищ можна віднести до типового патологічного процесу?

A.*Пухлину

B.Туберкульоз

C.Гіпертонічну хворобу

D.Непрохідність кишечника

E.Інфаркт міокарда

21.При стоматологічному огляді у пацієнта 37-ми років, було констатовано відсутність 1 лівого верхнього премоляру. За словами пацієнта, зуб було видалено два роки тому внаслідок парадонтозу. Яке явище спостерігається у пацієнта?

A.*Патологічний стан

B.Патологічна реакція

C.Патологічний процес

D.Хронічний процес

E.Ускладнення

22.Жінка, яка лічила зуби з приводу карієса, із-за ускладнення погодилася на видалення зуба. Що з патологічних явищ у жінки можна віднести до поняття “патологічний стан”?

A.* Відсутність зуба.

B.Підвищення температури.

C.Почервоніння.

D.Припухлість

E.Набряк

23.В инфекциооное отделение поступил больной с жалобами на боль в правом подреберье, общую слабость, желтушность кожи, обесцвеченный кал. Объективно: склеры и кожа желтушны, t-39oC, печень увеличена, кал ахоличный. Был поставлен диагноз "гепатит". Какой стадии болезни соответствуют описанные явления?

A.*Разгару болезни

B.Инкубационному периоду

C.Латентному периоду

D.Продромальному периоду

E.Исходу болезни

24.У чоловіка, якому 2 тижні назад були встановлені коронки зубів, десневі сосочки і

слизова оболонка десневого краю, гіперемовані, припухлі, кровоточать. Диагностован гінгівіт. Чим є гіперемія при гінгівіті у чоловіка?

A.*Симптом.

B.Синдром.

C.Нозологічна одиниця.

D.Типовий патологічнийй процес.

E.Патологічний стан.

25.Після перенесеного артриту у хворого обмежилась рухливість суглоба. Як називається такий вид патології?

A.*Патологічний стан

B.Патологічний процес

C.Хвороба

D.Патологічна реакція

E.Типовий патологічний процес

26.Відомо, що перебіг хвороби залежить від багатьох факторів, особливо від реактивності організму. Як називається повторний спалах хвороби після деякого періоду її відсутності?

A.* Рецидив

B.Ремісія

C.Ускладнення

D.Хронічна форма

E.Неповне видужання

27.В конце зимы студент, который в последнее время отмечал нервное перенапряжение, после переохлаждения заболел острым респираторным заболеванием. Что послужило причиной его заболевания?

A.*Патогенный возбудитель

B.Нервное перенапряжение

C.Переохлаждение

D.Нерациональное питание

E.Гиповитаминоз

28.Больному с пневмонией назначено комплексное лечение этиотропное, патогенетическое, симптоматическое. К этиотропным средствам фармакокоррекции относятся препараты, воздействующие на:

A.*Причину и условия развития заболевания

B.Причину развития заболевания

C.Условия, способствующие развитию заболевания

D.Причинно-следственные связи

E.Функцию больного органа

29.У чоловіка 36-ти років, який прибув на відпочинок в гори, на висоту більше 2000 м над рівнем моря, спостерігалось збільшення частоти дихання, тахікардія, незначне запаморочення, які нормалізувались через дві доби. Цей процес називається:

A.* Адаптація

B.Компенсація

C.Регенерація

D.Гальмування

E.Проліферація

30.У хворого Д., була проведена ектракція зуба. Яке з патологічних явищ відноситься до

поняття “ патологічний стан”?

A.*Відсутність зуба

B.Пульпіт

C.Карієс

D.Періодонтіт

E.Запалення

31.Пациент, находящийся на диспансерном учете с диагнозом – хронический гингивит (воспаление слизистой оболочки десен), обратился весной с жалобами на боли и кровоточивость десен. Как можно охарактеризовать такое состояние пациента?

A.*Рецидив

B.Осложнение

C.Ремиссия

D.Патологическая реакция

E.Патологическое состояние

32.У пациента с запущенным кариесом появились боли на горячую и холодную пищу, врач-стоматолог диагностировал пульпит (воспаление пульпы зуба). Как можно охарактеризовать такое состояние пациента?

A.*Осложнение

B.рецидив

C.Ремиссия

D.Патологическая реакция

E.Патологическое состояние

33.У пациента с диагнозом – хронический гингивит, при очередном осмотре у

врача-стоматолога не обнаружено воспалительных изменений слизистой оболочки десен. Как можно охарактеризовать такое состояние пациента?

A.*Ремиссия

B.Рецидив

C.Осложнение

D.Патологическая реакция

E.Патологический процесс

34.Хворий звернувся до лікаря з приводу фурункула (запалення волосяного фолікула) в ділянці спини. Гарячки, ознак інтоксикації у хворого немає. Це найбільш імовірно:

A.* Патологічний процес

B.Патологічний стан

C.Патологічна реакція

D.Хвороба

E.-

35.Хворий 5-ти років захворів гостро з підвищенням температури тіла до 38,2С, скаржиться на біль при ковтанні. В ротовій порожнині висипи яскраво червоного кольору, почервоніння ясен в ділянці фронтальних зубів верхньої щелепи. Яке нозологічне поняття характеризує стан хворого?

A.*Розпал хвороби

B.Рецедив хвороби

C.Загострення хвороби

D.Патологічна реакція

E.Період ремісії

36.Мужчина 60-ти лет вследствие длительного пребывания в мокрой одежде при низкой температуре окружающей среды заболел крупозной пневмонией. Какова причина возникновения такой формы воспаления лёгких?

A.Возраст

B.*Пневмококк

C.Снижение реактивности организма

D.Воздействие на организм низкой температуры

E.Воздействие на организм высокой влажности

37.У больного с признаками недостаточности митрального клапана в намнезе отмечались атаки ревматизма, сопровождавшиеся воспалительными явлениями в суставах. Какое из патологических явлений у данного больног относится к категории "патологического состояния"?

A.Артрит

B.Ревматизм

C.*Недостаточность митрального клапана

D.Воспаление суставов

E.Ревмокардит

38.Мужчина получил дозу облучения 30 Гр. У него наблюдается некротическая ангина, расстройства желудочно-кишечного тракта. В крови: анемия, лейкопения, тромбоцитопения. Какой период острой лучевой болезни у мужчины?

A.*Разгар болезни

B.Конец болезни

C.Первичных реакций

D.Мнимого благополучия

E.

39.У чоловіка 49-ти років, який 12 років тому хворів ревматичним міокардитом та ендокардитом, є недостатність мітрального клапану. Дослідження показали, що запального процесу зараз не має, хвилинний об’єм кровообігу достатній. Якому поняттю загальної нозології відповідає дана умова?

A.*Патологічний стан.

B.Патологічна реакція.

C.Патологічний процес.

D.Типовий патологічний процес.

E.Компенсаторна реакція.

БАРОТРАВМА. ДЕЙСТВИЕ НА ОРГАНИЗМ ЧЕЛОВЕКА ПОНИЖЕННОГО ЛИБО ПОВЫШЕННОГО АТМОСФЕРНОГО ДАВЛЕНИЯ.

1.При развитии горной болезни расстройства функций организма во многом зависят от степени чувствительности различных органов к недостатку кислорода во вдыхаемом воздухе. Укажите, какой из перечисленных органов либо систем имеет наибольшую чувствительность к недостатку кислорода ?

А.Скелетная мускулатура В.Эндокринная система С.*Головной мозг D.Тонкий кишечник E.Спинной мозг

2.При развитии горной болезни последовательно развиваются перечисленные ниже изменения показателей состояния жизненных функций организма. Какое из них носит собственно патологический характер?

A.Учащение дыхания

B.Углубление дыхания

C.Учащение пульса

D.*Гипоксемия

E.Гиподинамия и апатия

3.При высотной болезни возникает ряд изменений в организме. Какое из перечисленных явлений носит защитно-приспособительный характер?

A.Эйфория

B.Гипоксемия

C.Гипокапния

D.*Апатия и гиподинамия

E.Сгущение крови

4.При развитии горной болезни имеет место ряд последовательных изменений в организме. Укажите, какое из перечисленных изменений является первичным патогенетическим фактором развития данного вида патологии?

A.Гипокапния

B.*Гипоксемия

C.Одышка

D.Учащение сердечной деятельности

E.Снижение двигательной активности

5.Первые проявления горной болезни у здорового нетренированного к подъему человека имеют свой порог высоты 2,5 – 3 тыс. метров над уровнем моря. Что из перечисленного вероятнее всего обусловливает это явление?

A.Возникновение одышки

B.Усиление кровообращения

C.Увеличение числа эритроцитов

D.*Способность гемоглобина лучше присоединять кислород на допороговой высоте

E.Сдвиг кривой диссоциации гемоглобина в правую сторону

6.При развитии горной болезни имеет место последовательность изменений характера дыхания. Какой характер оно носит у ранее здорового человека в начале развития этого вида патологии?

A.Поверхностное частое

B.Поверхностное редкое

C.Периодическое

D.Редкое глубокое

E.*Частое глубокое

7.У альпиниста 27-ми лет на высоте 5 000 м над уровнем моря впервые во время сна изменился характер дыхания: после нескольких глубоких дыхательных движений наступает остановка дыхания, за которой снова возникают глубокие дыхательные движения и т.д. Какова наиболее вероятная причина изменения дыхания?

A.Снижение температуры воздуха

B.Снижение парциального давления СО2 в воздухе

C.*Снижение парциального давления О2 в воздухе

D.Увеличение скорости кровотока

E.Повышение кислородной емкости крови

8.У альпиниста во время восхождения на высоте 6 000 м над уровнем моря возникла эйфория, неадэкватная оценка обстановки, наблюдались галлюцинации. Какова главная причина развития этих признаков горной болезни?

A.Физическое перенапряжение

B.Снижение атомосферного давления

C.Снежная офтальмия

D.*Снижение парциального давления кислорода в воздухе

E.Расширение воздуха в лобных пазухах

9.У группы альпинистов на высоте 3 000 м был сделан анализ крови. При этом было

обнаружено снижение НСО3 до 15 ммоль/л (норма 22-26 ммоль/л). Каков механизм снижения НСО3 крови?

A.*Гипервентиляция

B.Снижение реабсорбции бикарбонатов в почках

C.Гиповентиляция

D.Усиление ацидогенеза

E.Cнижение аммониогенеза

10.Человек длительное время проживал в условиях высокогорья. Какое изменение в системе крови будет у него наблюдаться?

A.Увеличение диаметра кровеносных сосудов

B.Снижение количества лейкоцитов

C.Снижение количества эритроцитов

D.*Повышение количества эритроцитов

E.Снижение количества тромбоцитов

11.Альпинист на протяжении нескольких суток поднимался в гору. На высоте 5000 м стали беспокоить тахипноэ, тахикардия, боль в ушах, головная боль распирающего характера. Укажите возможную причину наблюдаемых явлений.

A.Повышение парциального давления кислорода в воздухе

B.Недостаточная вентиляция легких

C.Газовая эмболия

D.Снижение температуры воздуха

E.*Снижение барометрического давления

12.При изучении влияния низкого барометрического давления на белых крысах в барокамере на высоте, соответствующей 2 000 м над уровнем моря, у животных

отмечается тахипноэ, увеличение количества эритроцитов и концентрации гемоглобина. На какой высоте над уровнем моря отмечается явление „самозакипания” крови?

A.14 000 м

B.7 500 м

C.8 500 м

D.*19 000 м

E.10 000 м

13.Группу белых крыс поместили в камеру с низким парциальным содержанием кислорода и заставили бежать в тредбане с нарастающей скоростью. На протяжении всего исследования с помощью вживленных датчиков регистрировали частоту и силу сердечных сокращений. Через 30 минут после начала эксперимента у крыс возникла прогрессирующая сердечная недостаточность. Какова основная причина развития сердечной недостаточности у крыс в условиях возникающей гипоксии?

A.Снижение активности АТФ-азы

B.*Возникновение дефицита АТФ

C.Усиление синтеза АТФ

D.Активация АТФ-азы

E.Усиление ресинтеза АТФ

14.Какой из механизмов нарушения функций организма не включается при действии сниженного барометрического давления?

A.Механическое действие на стенки воздухоносных путей из-за градиента давления по обе их стороны

B.*Изменение осмотического давления крови

C.Уменьшение процента оксигемоглобина

D.Закипание биологических жидкостей

E.Газовая эмболия вследствие десатурации газов

15.При восхождении в гору у группы студентов наблюдались: тахипноэ, эйфория, тахикардия, увеличение в крови эритроцитов и гемоглобина. Какое из состояний со стороны крови сопровождает описанные выше явления?

A.Истинная эритремия

B.*Эритроцитоз

C.Мегалобластическая анемия

D.Лейкопения

E.Тромбоцитопения

16.У члена высокогорной экспедиции на высоте 6 км возникло головокружение, слабость. Альпинист потерял сознание, дыхание остановилось. Эти нарушения возникли вследствие:

A.Недостаточного поступления О2 в организм

B.*Чрезмерного выведения СО2 из организма

C.Недостаточного образования СО2 в тканях

D.Недостаточной утилизации О2 тканями

E.Недостаточного освобождения О2 гемоглобином

17.У туристов, которые поднялись на высоту 3000 м, дыхание стало частым и глубоким. Эти изменения являются следствием стимуляции:

A.Барорецепторов дуги аорты

B.*Хеморецепторов каротидных синусов

C.Механорецепторов легочных альвеол

D.Барорецепторов каротидного синуса

E.Миоцитов дыхательных мышц

18.У пилота на высоте 14 000 м случилась аварийная разгерметизация кабины.Какой вид эмболии у него развился?

A.*Газовая

B.Тромбоэмболия

C.Воздушная

D.Жировая

E.Эмболия инородным телом

19.У водолаза, проводившего работы на большой глубине, при быстром возвращении его в условия нормального атмосферного давления появилась боль в суставах, зуд кожи, нарушение зрения, потеря сознания. Как называется описанное явление ?

A.Баротравма

B.Состояние невесомости

C.*Болезнь декомпрессии

D.Синдром взрывной декомпрессии

E.Гипероксия

20.При подъеме на «высоту» в барокамере у крысы появилось частое дыхание,

тахикардия, снижение рО2 и повышение рСО2 в крови. Какая форма гипоксии имеет место в данном случае?

A.Гемическая

B.Циркуляторная

C.Тканевая

D.Анемическая

E.*Гипоксическая

21.При подъеме в горы на высоте 5 000 м у участников альпинистской группы появились жалобы на одышку, учащенное сердцебиение, головную боль, головокружение, шум в ушах. Какой из перечисленных факторов вызвал указанные явления?

A.*Гипоксемия

B.Гипокалиемия

C.Увеличение кислородной емкости крови

D.Лактацидемия

E.Гипотермия

22.Врач-исследователь в составе альпинистской экспедиции поднялся на высоту 6 000 м. На 3-й день пребывания у него появились признаки горной болезни: одышка, головная боль, потеря аппетита, общая слабость, цианоз. Какой тип гипоксии имеет место в данном случае?

A.Тканевая

B.Гемическая

C.Застойная

D.*Гипоксическая

E.Циркуляторная

23.Для человека существует строгое ограничение во времени пребывания на высоте более 8000 метров над уровнем моря без кислородных баллонов. Что является лимитирующим фартором для жизни в данном случае?

A.Уровень жидкости

B.Сила земного притяжения

C.*Парциальное давление кислорода в воздухе

D.Температура

E.Уровень ультрафиолетового излучения

24.После погружения водолаза на глубину 60 м у него появились симптомы нарушения функций центральной нервной системы – возбуждение, эйфория, ослабление внимания профессиональные ошибки. Эти симптомы связаны с токсичным действием на нейроны:

А.*Азота В. Кислорода

C.Углекислого газа

D.Аммиака

E.Лактата

25.При подъеме в горы на высоте 5000 метров у участников альпинистской группы появились жалобы на одышку, учащенное сердцебиение, головную боль головокружение, звон в ушах. Какой патогенетический фактор определяет указанные явления?

A.*Гипоксемия

B Гипокалиемия

C.Увеличение кислородной емкости крови

D.Лактацидемия

E.Гипернатриемия

26.При подъеме в горы у группы туристов возникли признаки горной болезни. Укажите, какой из названных ниже факторов играет основную роль в ее развитии?

A.Солнечная радиация

B.*Снижение парциального давления кислорода во вдыхаемом воздухе

C.Перепад дневной и ночной температур

D.Скорость набора высоты

E.Тяжелая физическая нагрузка

27.При погружении под воду на человека действует повышенное атмосферное давление (компрессия). В крови и тканях организма растворяется дополнительное количество газов (сатурация). Возникают нарушения деятельности ЦНС : легкое возбуждение, эйфория, затем наркоз и интоксикация. Под действием какого газа на организм возникают эти изменения?

A.*Азота

B.Кислорода

C.Углекислого газа

D.Водорода

E.Гелия

28.У альпініста під час сходження на висоті 6 тис. м над рівнем моря виникла ейфорія, неадекватна оцінка обстановки, спостерігались галюцинації. Яка головна причина у розвитку цих ознак гірської хвороби?

A.Снігова офтальмія.

B.Фізичне навантаження.

C.Пониження атмосферного тиску

D.*Пониження парціального тиску кисню у повітрі

E.Розширення повітря в лобних пазухах.

29.Альпініст на протязі кількох діб підіймався в гору. На висоті 5 000 метрів його стали непокоїти тахіпное, тахікардія, головний біль розпираючого характеру. Вкажіть можливі причини вказаних симптомів?

A.Газова емболія

B.Зниження барометричного тиску повітря

C.Недостатня вентиляція легень

D.*Зниження парціального тиску кисню в повітрі

E.Зниження температури повітря

30.При розгерметизації кабіни літака на висоті 19 км наступила миттєва смерть пілотів. Яка її причина?

A.*Закипання крові

B.Крововилив в головний мозок

C.Газова емболія судин серця

D.Кровотечі

E.Параліч дихального центра

ДЕЙСТВИЕ НА ОРГАНИЗМ ЧЕЛОВЕКА ИОНИЗИРУЮЩЕЙ РАДИАЦИИ. ЛУЧЕВАЯ БОЛЕЗНЬ

1.Работника АЭС доставили в клинику после одноразового облучения с жалобами на головную боль, потерю сознания, повышение температуры тела, слабость, рвоты, понос. В анализе крови – лейкоцитоз с лимфопенией. Какой период лучевой болезни наиболее вероятен у пациента?

A.Продромальный

B.*Первичных реакций

C.Латентный

D.Развернутой клинической картины

E.Мнимого благополучия

2.Во время ликвидации аварии на ЧАЭС рабочий получил дозу радиации 4 ГР. В анализе его крови обнаружено: эритроцитов 2,3х1012/л, ретикулоцитов – 0,02%, Hb –60 г/л, лейкоцитов – 3х109/л, в лейкоцитарной формуле отмечается лимфопения, тромбоцитов – 40х109/л. Для какого периода лучевой болезни характерны такие изменения?

A.Начального

B.Скрытого

C.*Разгара

D.Первичных реакций

E.Мнимого благополучия

3.Пациент, который находится в клинике по поводу острой лучевой болезни, жалуется на головную боль, расстройства сна, выраженную слабость, затруднение приема пищи. Во время обследования обнаружены воспаление языка и десен, некротическая ангина, на коже многочисленные кровоизлияния, кровь в моче и кале. Поражение какой ткани является ведущим в патогенезе расстройств в данном случае?

A.Костной

B.Железистого эпителия

C.Нервной

D.Лимфоидной

E.*Кроветворной

4.В эксперименте кролик получил рентгеновское облучение в дозе 500 Гр. Какие механизмы повреждения клеток при этом являются основными?

A.Ацидотические

B.Кальциевые

C.*Липидные

D.Электролитные

E.Протеиновые

5.В период разгара острой лучевой болезни у пациента отмечалась резкая слабость, холодный пот, бледность кожных покровов, тахикардия, боль в различных отделах кишечника, рвота, примесь крови в кале. Артериальное давление 70/40 мм рт.ст. Две недели тому назад больной получил дозу облучения 30 Гр. Какой форме лучевой болезни присущи такие симптомы?

A.*Кишечной

B.Токсемической

C.Костномозговой

D.Геморрагической

E.Церебральной

6.Крыса получила рентгеновское облучение в дозе 200 бер. Назовите наиболее вероятное изменение периферической крови, наблюдающееся через 10 часов после облучения.

A.Лейкопения, анемия

B.Анемия

C.Анемия, лейкопения, тромбоцитопения

D.*Нейтрофильный лейкоцитоз

E.Агранулоцитоз

7.В эксперименте на крысах проводилась сравнительная количественная оценка биологического действия различных видов излучения. Установлено, что наибольшая биологическая эффективность присуща альфа-излучению, протонам и быстрым нейтронам. Какой критерий считается главным при оценке относительной биологической эффективности (ОБЭ) ?

A.*Смертность, гематологические и морфологические изменения в тканях

B.Плотность ионизации и проникающая способность излучения

C.Природа излучения (корпускулярное или электромагнитное)

D.Доза и мощность полученной радиации

E.Вид излучения ( внешнее или внутреннее )

8.Кролик получил облучение в дозе 0,5 Гр от рентгеновской установки. В какой системе наблюдаются наиболее выраженные расстройства через 10 часов после облучения?

A.Кроветворной

B.*Нервной

C.Сердечно-сосудистой

D.Половой

E.Лимфоидной

9.При работе по ликвидации последствий аварии на АЭС работник получил дозу облучения 500 рентген. Жалуется на головную боль, тошноту, временами потерю сознания. Какие изменения со стороны лейкоцитов можно ожидать у больного через 10 часов после облучения?

A.Лимфоцитоз

B.Лейкопению

C.Агранулоцитоз

D.*Нейтрофильный лейкоцитоз

E.Лейкемию

10.В периоде разгара острой лучевой болезни у больного наблюдались лейкопения, тромбоцитопения, аутоинфекция, аутоинтоксикация, кровоточивость, повышение температуры тела. Для какой формы лучевой болезни характерна данная картина?

A.Кишечной

B.*Костно-мозговой

C.Токсемической

D.Церебральной

E.Геморрагической

11.В результате нарушения правил техники безопасности работник радиологического отделения областной больницы получил однократное облучение. Через 8 дней развились язвенно-некротические изменения в полости рта. При анализе крови обнаружена небольшая гипохромная анемия, выраженная лейкопения, значительная тромбоцитопения, снижение числа ретикулоцитов. Для какого периода лучевой болезни характерны описанные изменения?

A.Периода первичных реакций

B.*Периода развернутых клинических признаков

C.Скрытого периода

D.Периода мнимого благополучия

E.Исхода болезни

12.В результате повреждения одного из реакторов АЭС произошло вытекание радиоактивных продуктов. Люди, которые находились в зоне повышенной радиации, ориентировочно получили по 250-300 Р. Они были немедленно госпитализированы. Какие изменения состава крови будут характерны для потерпевших?

A.Нейтропения

B.Тромбоцитопения

C.Лейкопения

D.*Лимфопения

E.Анемия

13.Ликвидатор последствий аварии на ЧАЭС получил дозу облучения 5 Гр. Через неделю у него выявлен агранулоцитоз. Какой патогенетический механизм является ведущим в его возникновении?

A.Увеличение перехода гранулоцитов в ткани

B.Увеличение разрушения лейкоцитов

C.*Угнетение лейкопоэза

D.Нарушение выхода зрелых лейкоцитов из костного мозга

E.Развитие аутоиммунного процесса

14.При работе с радиоактивными веществам сотрудник вследствие аварии получил дозу общего облучения 4 Гр. Жалуется на головную боль, тошноту, головокружение. Какие изменения в составе крови можно ожидать у больного через 10 часов после облучения?

A.*Нейтрофильный лейкоцитоз

B.Лимфоцитоз

C.Агранулоцитоз

D.Нейтропению

E.Лейкопению

15.Мужчина 30-ти лет получил облучение дозой около 3 Гр. Какое изменение крови будет через 8 часов после облучения?

A.Анемия

B.Лейкопения

C.Гранулоцитопения

D.*Лимфопения

E.Тромбоцитопения

16.У ликвидатора последствий аварии на АЭС во время течения острой лучевой болезни возник гемморагический синдром. Что имеет наибольшее значение в патогенезе этого синдрома?

A.Уменьшение активности факторов свертывания крови

B.Повышение активности факторов фибринолиза

C.Нарушение структуры стенки сосудов

D.*Тромбоцитопения

E.Повышение активности факторов системы противосвертывания крови

17.У больного костномозговой формой лучевой болезни определили такие изменения

гемограммы: лейк.- 2*109/л, лимфопения, эр.- 3,0*1012/л, Hb52 г/л

тромб.- 105*109/л, свертывание крови снижено. Какой стадии болезни отвечают такие изменения?

A.*Разгару болезни

B.Латентному периоду

C.Продромальному периоду

D.Концу болезни

E.Рецидиву

18.Во время работы, связанной с ликвидацией последствий аварии на АЭС, работник получил дозу ионизирующего излучение 6 Гр.( 600 рентг.). Жалуется на общую слабость, тошноту, головокружение, лабильность артериального давления и пульса, кратковременный лейкоцитоз с лимфопенией. Для какого периода острой лучевой болезни характерны выше перечисленные признаки?

A.*Начального

B.Разгара

C.Скрытого

D.Завершения

E.Отдаленных последствий

19.В результате повреждения одного из реакторов АЭС произошло витекание радиоактивных продуктов. Люди, которы находились в зоне повышенной радиации, ориентировачно получили по 250-300 Р. Их немедленно госпитализировали. Какие изменения крови будет характерно в этот период?

A.*Лимфопения

B.Лейкопения

C.Анемия

D.Тромбоцитопения

E.Нейтропения

20. Женщина 52 г., больная раком нижней челюсти, прошла курс лучевой терапии. Размер опухоли уменьшился. Какой из приведенных механизмов повреждения клетки наиболее обусловливает эффективность лучевой терапии?

A.*Образование свободных радикалов

B.Гипертермия

C.Лизис NK-клетками

D.Тромбоз сосудов

E.Мутагенез

21.Пацієнт хворий на рак шлунка пройшов декілька курсів радіотерапії. Функція якої системи в першу чергу порушується після дії на організм іонізуючого випромінювання?

A.Травної

B.Нервової

C.*Крові

D.Сечовидільної

E.Дихальної

22.У кролика після опромінення спостерігається III період кістково-мозкової форми гострої променевої хвороби. Ураження якої тканини є провідним у патогенезі розладів при цьому?

A.*Кровотворної

B.Кісткової

C.Нервової

D.Епітелію статевих залоз

E.Залозистого епітелію

23.При воздействии радиоактивного излучения в дозе 5 Гр был поврежден красный костный мозг. Чем определяется чувствительность красного костного мозга к ионизирующему излучению?

A.*Интенсивностью деления клеток

B.Высоким уровнем свободных радикалов в клетках тканей

C.Высоким уровнем перекисей в клетках тканей

D.Наличием радиосенсибилизирующих веществ в клетках

E.Деструктивным действием радиотоксинов на синтез ДНК

24.Під час аварії на атомному підводному човні підводники були опромінені. Первинна іонізація яких молекул має найбільше значення в розвитку променевої хвороби?

A.*Води

B.Структурних білків

C.Ліпідів

D.Нуклеїнових кислот

E.Ферментів

25.В развитии лучевого повреждения выделяют непрямое действие радиации. Что является при этом первичным звеном патогенеза?

A.*Образование свободных радикалов

B.Окисление белков

C.Окисление липидов

D.Повреждение ДНК

E.Накопление радиотоксинов

26.При изучении сравнительной радиочувствительности тканей была обнаружена не одинаковая их чувствительность к действию ионизирующего излучения. Укажите, какая из перечисленных тканей является наиболее радиочувствительной?

A.* Кроветворная

B.Хрящевая

C.Костная

D.Мышечная

E.Нервная

27.У кроля після опромінення спостерігається III період кістково-мозкової форми гострої променевої хвороби. Ураження якої тканини є провідним у патогенезі розладів при цьому?

A.Кісткової.

B.*Кровотворної.

C.Нервової.

D.Епітелію статевих залоз.

E.Залозистого епітелію.

28.Ліквідатор наслідків аварії на ЧАЕС отримав дозу іонізуючого опромінення 6 Гр. Які зміни лейкоцитарної формули слід очікувати через 10 днів?

A.* Агранулоцитоз

B.Лімфоцитоз

C.Лейкоцитоз з лімфоцитопенією

D.Базофілію

E.Єозінофілію

29.Через 8 дней после облучения у ликвидатора ЧАЭС развились язвенно-некротические

изменения в полости рта. В анализе крови: Эр-3,2*1012/л, ретикулоцитов 0,01%, Hb60 г/л, лейкоцитов 2,3*109/л, тромбоцитов 50 тыс/л Для какого периода лучевой болезни характерны описанные изменения?

A.*Периода развернутых клинических признаков

B.Периода первичных реакций

C.Скрытого периода

D.Периода мнимого благополучия

E.Исхода болезни

30.Ураження хворого однократною дозою іонізуючого випромінювання спричинило розвиток кістково-мозкової форми променевои хвороби. Які патологічні прояви з боку крови будуть характерними в період удаваного благополуччя?

A.*Наростаюча лімфопенія, лейкопенія

B.Перерозпридільний лейкоцитоз, лімфоцитоз

C.Анемія, лейкопенія

D.Тромбоцитопенія, анемія

E.Тромбоцитопенія, лейкоцитоз

РЕАКТИВНОСТЬ ОРГАНИЗМА. БАРЬЕРНЫЕ ПРИСПОСОБЛЕНИЯ. ДИАТЕЗЫ

1.Мальчик на втором году жизни начал часто болеть респираторными заболеваниями, поражениями кожи. Даже незначительные поражения осложняются длительно протекающим воспалением. Реакция Манту в пределах нормы. Наследственное снижение функциональной активности какой клеточной популяции лежит в основе синдромов?

A.*В-лимфоцитов

B.Т-лимфоцитов

C.Нейтрофилов

D.Эндотелиоцитов

E.Макрофагов

2.После приема алкоголя и наркотиков у человека участились заболевания кожи воспалительного характера. Чем объясняется повышенная заболеваемость этого человека?

A.Повреждением гистогематического барьера

B.Повреждением лейкоцитов

C.*Повреждением печени

D.Нарушением метаболизма

E.Нарушением рН кожи

3.В условиях подъема в горы у человека возникла одышка, тахикардия, эйфория. Наблюдаемые изменения являются проявлением реактивности:

A.Специфической

B.*Неспецифической

C.Групповой

D.Дизергической

E.Гиперергической

4.Больному радикулитом была сделана новокаиновая блокада, после которой у него потемнело в глазах, артериальное давление снизилось до 90/50 мм рт.ст., дыхание участилось до 24 в мин. Состояние больного является проявлением реактивности:

A.Физиологической

B.Гипергической

C.Нормергической

D.Неспецифической

E.*Гиперергической

5.После травмы одного глаза у больного постепенно начало падать зрение в другом глазу. Каким механизмом можно объяснить это явление?

A.Снижением резистентности организма

B.Инфекцией

C.*Повреждением гистогематического барьера

D.Иммунодефицитом

E.Иммунодепрессией

6.У ребенка на протяжении первого года жизни наблюдаются частые бактериальные инфекции. Аллергическая реакция на туберкулин положительная. С чем связано состояние иммунодефицита?

A.Врожденной Т-клеточной недостаточностью

B.Приобретенным иммунодефицитом

C.*Врожденной В-клеточной недостаточностью

D.Врожденным дефектом Т-супрессоров

E.Врожденным комбинированным (тотальным) иммунодефицитом

7.После проведения курса лечения антибиотиками у больного пневмонией в стадии выздоровления появилась диаррея, причиной которой стал дисбактериоз,вызванный кишечной палочкой, и генерализованный кандидоз. В дальнейшем у пациента отмечались частые острые респираторные заболевания. Кожная аллергическая проба на туберкулин отрицательная, в крови снижено содержание g-глобулинов. Как расценивать состояние больного?

A.Врожденный Т-клеточный иммунодефицит

B.Гипогаммаглобулинемия

C.Агаммаглобулинемия

D.Врожденный комбинированный (тотальный) иммунодефицит

E.*Приобретенное иммунодефицитное состояние

8.У ВИЧ-инфицированного человека отмечаются частые простудные заболевания. Какие клетки первично поражает вирус иммунодефицита человека?

A.*Т-лимфоциты хелперы

B.В-лимфоциты

C.Плазмоциты

D.Т-лимфоциты киллеры

E.Т-лимфоциты супрессоры

9.У больного наблюдается швейцарский тип иммунодефицита. Его наиболее вероятным признаком будет:

A.Трансформация В-лимфоцитов

B.Лимфопения

C.Дефицит Т- и В-лимфоцитов

D.Снижение числа плазматических клеток

E.*Моноцитопения

10.После прививки (КДС) погиб мальчик 11 лет. При изучении истории развития установлено, что у него периодически возникала экзема и тромбоцитопения. Больной погиб вследствие наличия у него:

A.Дефицита системы Т-лимфоцитов

B.Гемофилии

C.*Комбинированного иммунодефицита

D.Дефицита системы В-лимфоцитов

E.Тромбоцитопении

11.Больной жалуется на снижение массы тела, диаррею, длительную лихорадку, длительный кашель, дерматит. При опросе сообщил, что полгода тому назад имел сомнительную половую связь. Назовите, какие клетки поражены в первую очередь возбудителем данного заболевания?

A.Т-киллеры

B.*Т-хелперы

C.Т-супрессоры и В-киллеры

D.Т-8-эффекторы

12.Известно, что при некоторых патологических процессах реактивность организма изменяется противоположно изменению его резистентности. При каких патологических процессах либо заболеваниях возможна такая ситуация?

A.*Шоке

B.Лихорадке

C.Воспалении

D.Постгеморрагической анемии

E.Артериальной гипертензии

13.У ребенка пяти лет, часто болеющего респираторными заболеваниями, отмечаются экзематозные явления после приема некоторых пищевых продуктов, склонность к затяжному течению воспалительных процессов. Какой вид диатеза можно предположить в данном случае?

A.Лимфатико-гипопластический

B.Астенический

C.Нервно-артритический

D.Геморрагический

E.*Экссудативно-катаральный

14.Больного К., 48лет, на протяжении длительного времени беспокоят частые грибковые заболевания ног и гнойничковая сыпь на поверхности туловища. В анамнезе злоупотребление алкоголем. Что способствовало реализации причины заболевания в данном случае?

A.Повреждение гематоэнцефалического барьера

B.*Снижение барьерной функции печени

C.Первичный иммунодефицит

D.Аллергия

E.Хроническая интоксикация

15.У пожилого человека отмечается повышенная чувствительность к инфекциям, в особенности активизация хронической вирусной инфекции, заболевание сосудов, в том числе атеросклеротического типа, пернициозная анемия. Изменения в какой системе при старении организма сопровождается описанными явлениями?

A.Эндокринной

B.Нервной

C.*Иммунной

D.Соединительной ткани

E.Кровообращения

16.У матери, страдающей алкоголизмом, родился ребенок. В процессе его роста и развития были обнаружены признаки отставания в физическом и умственном развитии. К патологии какого характера можно отнести данный случай?

A.Наследственной

B.Хромосомной

C.Приобретенной

D.*Врожденной

E.Перинатальной

17.Неспецифические факторы защиты полости рта от проникновения патогенных микроорганизмов играют важную роль в общей системе физиологической устойчивости организма. Какой из перечисленных компонентов в полости рта является важнейшим фактором неспецифической защиты?

A.Комплемент

B.Фагоцитоз

C.В-лизины

D.Пропердин

E.*Лизоцим

18.У больного при объективном осмотре обнаружены: тонкая фигура, большой череп; сильно развитая лобная часть лица, короткие конечности. Для какого конституционального типа это характерно?

A.*Респираторного

B.Мускульного

C.Дигестивного

D.Церебрального

E.Смешанного

19.У ребенка, часто болеющего ангинами и фарингитами, отмечается увеличение лимфоузлов и селезенки. Внешний вид характеризуется пастозностью и бледностью, мышечная ткань развита слабо. В крови наблюдается лимфоцитоз. Как называется такой вид диатеза?

A.Экссудативно-катаральный

B.Астенический

C.Геморрагический

D.*Лимфатико-гипопластический

E.Нервно-артрический

20.При воспалительных процессах у человека в крови увеличивается количество лейкоцитов. Эта закономерность является проявлением:

A.*Адаптации

B.Регенерации

C.Репарации

D.Перераспределения лейкоцитов

E.Дегенерации

ИММУНОПАТОЛОГИЧЕСКИЕ ПРОЦЕССЫ. АЛЛЕРГИЯ. ИММУНОДЕФИЦИТНЫЕ СОСТОЯНИЯ.

1.У хирурга возник контактный дерматит, который проявился воспалительными повреждениями кожи рук, обостряющимися после мытья рук при подготовке к операции. К какому типу аллергических реакций относится эта форма патологии?

A.Немедленного типа

B.Анафилаксии

C.Атопических болезней

D.Атипичного

E.*Замедленного типа

2.Больному острой пневмонией внутримышечно был введен пенициллин, после чего состояние больного резко ухудшилось: появилась одышка, больной покрылся холодным потом и утратил сознание. Пульс – 140/мин, слабого наполнения. Какое осложнение наиболее вероятно возникло у больного?

A.Тромбоэмболия легочной артерии

B.*Медикаментозный анафилактический шок

C.Обморок

D.Инфекционно-токсический шок

E.Сывороточная болезнь

3.Больной острой бронхопневмонией была сделана внутримышечная инъекция пенициллина. Через 30 минут у нее появился цианоз губ, гиперемия лица, тахикардия, снижение АД. Врач сделал вывод, что у нее развился анафилактический шок. Какое из перечисленных биологически активных веществ является патогенетическим фактором анафилактического шока в данном случае?

A.Адреналин

B.Лизосомальные ферменты

C.*Гистамин

D.Лимфокины

E.Интерлейкины

4.У больной бронхиальной астмой в крови в три раза повышен уровень брадикинина. Активация какой из перечисленных ниже систем наиболее вероятно вызвала повышение количества брадикинина в данном случае?

A.Системы комплемента

B.Фактора Хагемана

C.*Протеолитических ферментов крови

D.Протеолитических ферментов тканей

E.Лизосомальных факторов

5.При проведении медицинского обследования детям была поставлена реакция Манту. У двоих детей она оказалась положительной. Наличием каких из перечисленных специфических факторов обусловлена положительная реакция?

A.Антител

B.Лейкоцитов

C.Эритроцитов

D.*Т-лимфоцитов

E.В-лимфоцитов

6.После введения ампициллина больная потеряла сознание, АД снизилось до 80/40 мм рт.ст., ЧСС – 90/мин, ЧД – 24/мин. Как называется первая стадия патогенеза развившегося у больной состояния?

A.*Иммунологическая

B.Стадия функциональных и структурных расстройств

C.Патохимическая

D.Биохимическая

E.Патофизиологическая

7.На протяжении последних лет у мужчины с началом цветения тополя начинается зуд и гиперемия глаз, носа, риноррея, покашливание, мелкое уртикарное высыпание открытых частей тела. При обследовании обнаружено резкое повышение уровня IgE. Аллергологом назначена специфическая гипосенсибилизирующая терапия. Каким образом проводят это лечение?

A.Введением хлористого кальция

B.Введением антигистаминных препаратов

C.*Многократным введением малых доз аллергена

D.Однократным введением большой дозы аллергена

E.Введением гормонов коры надпочечников

8.После травмы правого яичка через месяц у мужчины появилась боль в левом яичке. Объективно: кожа над яичком красного цвета, припухлая, яичко болезненно при дотрагивании, плотное. Какой процесс является причиной этих изменений?

A.Парааллергия

B.Феномен Артюса

C.Гетероаллергия

D.*Аутоаллергия

E.Инфекционной процесс

9.После 15 лет работы на производстве красок у женщины возник контактный дерматит верхних конечностей, шеи и лица (гиперемия, высыпания, пастозность). К какому из перечисленных ниже состояний относится это заболевание?

A.Аллергической реакции немедленного типа

B.Иммунодефициту В-системы

C.*Аллергической реакции замедленного типа

D.Поздней гипогаммаглобулинемии

E.Иммунодефициту Т-системы

10.У женщины отмечается пастозность век, губ, шеи, слизистой оболочки языка после того, как она съела апельсин. Ранее на апельсины возникали высыпания на коже, зуд. Какой патогенетический механизм является ведущим в развитии отека у женщины?

A.Повышение гидродинамического давления крови в капиллярах

B.Нарушение лимфооттока

C.*Повышение проницаемости капилляров

D.Снижение онкотического давления крови

E.Повышение онкотического давления тканевой жидкости

11.У больного ночью возник приступ одышки с выраженным затруднением дыхания, который был слышен в соседней комнате. Дыхание осуществлялось с помощью вспомогательной мускулатуры. Бронходилятаторы облегчили клиническое состояние больного. Обнаружена положительная аллергическая реакция к тополиному пуху. К

какому из перечисленных типов реакций гиперчувствительности относится данный случай?

A.Цитотоксической реакции немедленного типа

B.*Гуморальной реакции немедленного типа, обусловленной иммуноглобилинами типа IgE

C.Реакции иммунных комплексов

D.Гуморальной реакции немедленного типа, обусловленной иммуноглобулинами типа IgG

E.Клеточной реакции гиперчувствительности замедленного типа

12.Мужчина закапал в оба глаза капли, которые содержат антибиотик. Через несколько минут появился отек век, губ, щек, кашель. Для какого из перечисленных состояний характерна данная клиническая картина?

A.Анафилактического шока

B.*Отека Квинке

C.Коллаптоидного состояния

D.Приступа бронхиальной астмы

E.Идиосинкразии

13.Через неделю после введения с профилактической целью 3000 ед. противостолбнячной сыворотки по Безредко у больного появились признаки сывороточной болезни. По какому типу аллергической реакции наиболее вероятно протекает данное заболевание?

A.Как аутоаллергическое заболевание

B.По 1 типу аллергии – реагиновому

C.По П типу аллергии – цитотоксическому

D.*По Ш типу аллергиииммунокомплексному

E.По клеточно-опосредованному

14.У больного аллергией возник отек Квинке (генерализованный отек мягких тканей тела). Какой из патогенетических факторов отека является пусковым в данном случае?

A.Снижение внутритканевого гидростатического давления

B.Снижение онкотического давления плазмы крови

C.*Повышение проницаемости стенок капилляров

D.Повышение гидродинамического давления в капиллярах

E.Повышение осмотического давления в тканях

15.У больного через 48 часов после применения мази для натирания «Эфкамон» на коже плеча и предплечья появилось покраснение, сыпь в виде пузырьков, что свидетельствовало о развитии аллергической реакции. Определите тип этой реакции:

A.Цитотоксический

B.Иммунокомплексный

C.*Гиперчувствительность замедленного типа

D.Анафилактический

E.Псевдоаллергический

16.У больного 15-ти лет при применении аспирина возник приступ бронхиальной астмы. Какое биологически активное вещество спровоцировало бронхоспазм в данном случае?

A.Брадикинин

B.Простагландины

C.*Лейкотриены

D.Адреналин

E.Катехоламины

17.При обследовании больного было выявлено недостаточное количество иммуноглобулинов. Какие клетки иммунной системы продуцируют иммуноглобулин?

A.Т –киллеры

B.*Плазматические

C.Т- хелперы

D.Макрофаги

E.Микрофаги

18.Через 2 часа после переливания аллогенной плазмы у больного с ожогами появились боли в суставах, пояснице, геморрагические высыпания на коже, повысилась температура тела. Какая из аллергических реакций имеет место в данном случае?

A.Аутоиммунный васкулит

B.Анафилаксия

C.Отек Квинке

D.Крапивница

E.*Сывороточная болезнь

19.Женщину 32-х лет ужалила оса. На коже левой щеки ( на месте укуса ) – отек и гиперемия. Какой механизм отека является ведущим в данном случае?

A.Снижение онкотического давления крови

B.*Повышение проницаемости капилляров

C.Затруднение лимфооттока

D.Повышение гидродинамического давления крови в капиллярах

E.Повышение онкотического давления тканевой жидкости

20.У ВИЧ-инфицированного человека наблюдаются частые простудные заболевания. Какие клетки первично поражает вирус иммунодефицита человека?

A.*Т-лимфоциты хелперы

B.В-лимфоциты

C.Плазмоциты

D.Т-лимфоциты киллеры

E.Т-лимфоциты супрессоры

21.У больного ребенка выявлено повышенное количество эозинофилов. Это может указывать на:

A.Потерю большого количества крови

B.*Аллергический процесс

C.Потерю большого количества железа

D.Острый воспалительный процесс

E.Хронический воспалительный процесс

22.У ребенка на протяжении первого года жизни наблюдаются частые бактериальные инфекции. Аллергическая реакция на туберкулин положительная. С чем связано иммунодефицитное состояние?

A.Врожденной Т-клеточной недостаточностью

B.Приобретенным иммунодефицитом

C.*Врожденной В-клеточной недостаточностью

D.Врожденным дефектом Т-супрессоров

E.Врожденным комбинированным (тотальным) иммунодефицитом

23.У женщины 45-ти лет в период цветения трав появилось острое воспалительное заболевание верхних дыхательных путей и глаз: гиперемия, отек, слизистые выделения. Какой вид лейкоцитоза будет наиболее характерным при этом?

A.Базофилия

B.Нейтрофилия

C.Лимфоцитоз

D.*Эозинофилия

E.Моноцитоз

24.У мужчины 25-ти лет на приеме у стоматолога через несколько минут после промывания рта раствором фурациллина возник значительный отек губ. Какой тип аллергической реакции наблюдался в данном случае?

A.Иммунокомплексный

B.Гиперчувствительность замедленного типа

C.Цитолитический

D.*Анафилактический

E.Стимулирующий

25.Мужчина 25-ти лет жалуется на часто возникающие воспалительные заболевания различной локализации. Проба на ВИЧ-инфекцию оказалась положительной. Какой из указанных типов клеток наиболее существенно поражается ВИЧ?

A.Плазматические клетки

B.Киллеры

C.*Хелперы

D.Нейтрофильные гранулоциты

E.Моноциты

26.У пациента после пересадки чужеродного почечного трансплантата развилась реакция отторжения. Какие основные эффекторные клетки участвуют в данной иммунологической реакции?

A.*Т-лимфоциты – цитотоксические

B.В-лимфоциты

C.Т-лимфоциты – супрессоры

D.Т-лимфоциты – хелперы

E.Плазмоциты

27.У больного после парентерального введения антибиотика через 25 минут отмечались: тошнота, слабость, боли в животе, сердцебиение, затруднение дыхания, на коже появились высыпания в виде волдырей. Какая стадия аллергической реакции отмечалась у больного?

A.Иммунологическая

B.Сенсибилизации

C.Биохимическая

D.*Патофизиологическая

E.Патохимическая

28.Известно, что бронхиальная астма у больных развивается по механизму гиперчувствительности немедленного типа, который последовательно включает в себя 3 стадии:

A.Патофизиологическую, иммунологическую, патохимическую

B.Патохимическую,иммунологическую, патофизиологическую

C.*Иммунологическую, патохимическую, патофизиологическую

D.Патохимическую, патофизиологическую, иммунологическую

E.Патофизиологическую, патохимическую, иммунологическую

29.Больному при удалении зуба для обезболивания был введен раствор новокаина. Через несколько минут у него упало АД, произошла потеря сознания, возникла одышка, судороги. Каков механизм возникновения анафилактического шока?

A.Токсическое действие новокаина

B.*Сенсибилизация к новокаину

C.Десенсибилизация организма

D.Аутоаллергическое состояние

E.Парааллергия

30.Больному для обезболивания при удалении кариозного зуба врачом-стоматологом был введен раствор новокаина. Через несколько минут у больного появились симптомы анафилактического шока: падение АД, учащение дыхания, потеря сознания, судороги. К какому типу реакций можно отнести это состояние?

A.*Гиперчувствительности немедленного типа

B.Реакции цитолиза или цитотоксического действия

C.Реакции типа феномена Артюса

D.Реакции замедленной гиперчувствительности

E.Стимулирующих аллергических реакций

31.У новорожденных крысят в эксперименте была удалена вилочковая железа. При этом развилась болезнь, которая характеризуется резким снижением в крови лимфоцитов, развитием инфекций, спленомегалией, остановкой роста и летальным исходом. Какое нарушение функции иммунной системы при этом наблюдается?

A.Комбинированный дефект Т- и В-лимфоцитов

B.Гиперфункция системы В-лимфоцитов

C.Гиперфункция системы Т-лимфоцитов

D.Недостаточность системы В-лимфоцитов

E.*Недостаточность системы Т-лимфоцитов

32.У медсестры манипуляционного кабинета со стажем работы 20 лет развился контактный дерматит верхних конечностей. К какому типу иммунных нарушений относится данное заболевание?

A.Первичному иммунодефициту

B.*Аллергической реакции замедленного типа

C.Аллергической реакции немедленного типа

D.В-клеточному иммунодефициту

E.Т-клеточному иммунодефициту

33.У больного наблюдается синдром Ди Джорджи, в основе которого лежит гипоплазия вилочковой железы. К какой форме иммунной патологии относится это заболевание?

A.Дефицит макрофагов

B.Приобретенному иммунодефициту в системе Т-лимфоцитов

C.*Врожденному иммунодефициту в системе Т-лимфоцитов

D.Врожденному иммунодефициту в системе В-лимфоцитов

E.Приобретенному иммунодефициту в системе В-лимфоцитов

34.После проведения прививки КДС мальчик 11-ти лет погиб при клинической картине скарлатины. При изучении истории развития ребенка установлено, что у него

периодически отмечались петехиальные высыпания на коже рук и ног, экзема. Больной погиб вследствие наличия у него:

A.Дефицита системы В-лимфоцитов

B.Дефицита системы Т-лимфоцитов

C.Тромбоцитопении

D.*Комбинированного иммунодефицита

E.Гемофилии

35.Больному ввели в мышцу 500 тыс.ед.стрептомицина. Через 5 минут он почувствовал тошноту, слабость, боли в животе, сердцебиение, затруднение дыхания, на коже появились высыпания в виде волдырей. Какая стадия аллергической реакции отмечается у больного?

A.Патохимическая

B.Биохимическая

C.Иммунологическая

D.Сенсибилизации

E.*Патофизиологическая

36.Новорожденный ребенок был в контакте с больным корью и не заболел. Что лежит в основе этого явления?

A.Приобретенный иммунитет

B.Несовершенство терморегуляции

C.Несовершенство иммунной системы

D.Особенности степени развития ЦНС

E.*Врожденный иммунитет

37.После укуса пчелы на участке ткани предплечья у больного появилась резкая боль, гиперемия, отек. Спустя 10 минут состояние больного резко ухудшилось: появилась резкая бледность кожи, частый пульс, холодный пот. Артериальное давление упало до 80/50 мм рт.ст. Какой патологический процесс наблюдался у данного больного?

A.*Анафилактический шок

B.Коллапс

C.Острая гипотензия

D.Атопическая аллергия

E.Кардиогенный шок

38.У больного через 9 суток после введения лечебной сыворотки появилась крапивница, зуд кожи, отек ее и слизистых оболочек, припухание лимфатических узлов. Какое заболевание развилось?

A.Поллиноз

B.*Сывороточная болезнь

C.Феномен Шварцмана

D.Феномен Овери

E.Отек Квинке

39.У больного диагностирован тиреотоксикоз. В крови найдены антитиреоидные тела. Какой тип аллергической реакции по Кумбсу и Джеллу наблюдается при развитии этого заболевания?

A.Иммунокомплексный

B.*Стимулирующий

C.Анафилактический

D.Цитотоксический

E.Гиперчувствительность замедленного типа

40.Больному с обширными ожогами сделали пересадку донорской кожи. Но на 8 сутки трансплантат отек, изменился его цвет и на 11 сутки он начал отторгаться. Какие клетки принимают в этом участие?

A.В-лимфоциты

B.Эритроциты

C.Базофилы

D.*Т-лимфоциты

E.Эозинофилы

41.Через месяц после протезирования зубов пациент обратился к стоматологу с жалобами на покраснение и отек слизистых полости рта. Установлен диагноз: аллергический стоматит. Какой тип аллергической реакции по Джеллу и Кумбсу лежит в основе этого заболевания?

A.Реагиновый

B.Иммунокомплексный

C.Цитотоксический

D.Стимулирующий

E.*Гиперчувствительность замедленного действия

42.Животному, сенсибилизированному туберкулином, внутрибрюшинно введен туберкулин. Через 24 часа при лапаратомии выявлены венозная гиперемия и отек брюшины. В мазках-оттисках с брюшины большое количество лимфоцитов и моноцитов. Какой вид воспаления наблюдается у животного?

A.Гнойное

B.Серозное

C.*Аллергическое

D.Фибринозное

E.Асептическое

43.У ребенка, больного дифтерией, через 10 дней после введения антитоксической противодифтерийной сыворотки появились высыпания на коже, которые сопровождались сильным зудом, повысилась температура тела до 380С, появились боли в суставах. Какова причина этих явлений?

A.Атопия

B.Анафилактическая реакция

C.Контактная аллергия

D.*Сывороточная болезнь

E.Гиперчувствительность замедленного действия

44.У больной с клиническими признаками иммунодефицита количество и функциональная активность Т- и В- лимфоцитов не изменены. При обследовании на молекулярном уровне обнаружен дефект, при котором нарушена функция антигенпрезентации к иммунокомпетентным клеткам. Дефект каких клеток наиболее вероятен?

A.Фибробласты, Т-лимфоциты, В-лимфоциты

B.Т-лимфоциты, В-лимфоциты

C.*Макрофаги, моноциты

D.NK-клетки

E.0-лимфоциты

45.Больная 27 лет закапала в глаза капли, в состав которых входит пенициллин. Через несколько минут появились зуд и жжение кожи, отек губ и век, кашель со свистом, стало снижаться артериальное давление. Какие иммуноглобулины принимают участие в развитии данной аллергической реакции?

A.*IgЕ и IgG

B.IgА и IgМ

C.IgG и IgD

D.IgМ и IgG

E.IgМ и IgD

46.У ребенка вокруг царапины на коже возникли признаки воспаления: боль, покраснение, отек как признаки немедленной гиперчувствительности. Какие клетки крови обуславливают эти изменения?

A.Нейтрофилы

B.Лимфоциты

C.Моноциты

D.*Базофилы

E.Эозинофилы

47.Больной пневмонией получает бензилпенициллина натриевую соль в дозе 500 000 ЕД 6 раз в день. После очередного введения лекарственного средства возникла лихорадка, потеря сознания, судороги. Что развилось у больного?

A.Идиосинкразия

B.*Анафилактический шок

C.Кумуляция

D.Толерантность

E.Тахифилаксия

48.При подозрении на туберкулез больному ребенку сделали пробу Манту. Через 24 часа в месте введения аллергена возникли припухлость, гиперемия и болезненность. Какие основные компоненты определяют эту реакцию организма?

A.Макрофаги, В-лимфоциты и моноциты

B.Плазматические клетки, Т-лимфоциты и лимфокины

C.Гранулоциты, Т-лимфоциты и IgG

D.В-лимфоциты, IgМ

E.*Мононуклеары, Т-лимфоциты и лимфокины

49.У больного с периодическими приступами удушья, которые возникают при вдыхании разных ароматических веществ, диагностирована бронхиальная астма. Определяется повышение IgЕ. Для какого типа реакций это характерно?

A.Гиперчувствительности замедленного действия

B.Цитотоксического

C.Иммунокомплексного

D.Аутоиммунного

E.*Анафилактического

50. К врачу обратилась женщина с жалобами на то, что в весенний период у нее появляется насморк, осиплость голоса, покраснение век. Какой тип аллергической реакции по Джеллу и Кумбсу развивается в этом случае?

А. Иммунокомплексный В. Гиперчувствительности замедленного типа С. * Анафилактический

D.Стимулирующий Е. Цитооксический

51.Через несколько минут поле повторного введения пенициллина у больного появилась одышка, онемение языка, головокружение, гиперемия, а потом бледность кожи. Что обусловило такое тяжелое состояние больного?

A.Бронхиальная астма

B.Острый гломерелонефрит

C.Сывороточная болезнь

D.*Анафилактический шок

E.Гемолитическая анемия

52.У больного наблюдается аллергическая реакция, сопровождающаяся зудом, отеками и покраснением кожи. Концентрация какого биогенного амина повысилась в тканях?

A.Серотонина

B.Дофамина

C.*Гистамина

D.Гаммааминомасляной кислоты

E.Триптамина

53.При переливании крови у пациента начал развиваться внутрисосудистый гемолиз эритроцитов. Гипечувствительность какого типа развиласт у пациента?

A.Гиперчувствительность III типа ( иммунокомплексная)

B.Гиперчувствительность IV типа ( клеточная цитотоксичность)

C.*Гиперчувствтельность II типа ( антителозависимая)

D.Гиперчувствительность V типа ( гранулематоз)

E.Гиперчувствительность I типа ( анфилактическая)

54.У ребенка 10-ти лет поставлена проба Манту (с туберкулином). Через 48 часов на месте введения туберкулина появилась папула размером до 8 мм в диаметре. Реакция гиперчувствительности какого типа развилась после введения туберкулина?

A.*Реакция гиперчувствительности IV тип

B.Реакция типа сывороточной болезни

C.Реакция гиперчувствительности III типа

D.Атопическая реакция

E.Реакция типа феномена Артюса

55. Ребенку после анализа иммунограммы поставили диагноз первичный иммунодефицит гуморального звена иммунитета. Какая из причин может привести к развитию первичного иммунодефицита в организме ребенка?

A.Нарушения реактивности и резистентности организма

B.Токсическое повреждение В-лимфоцитов

C.Нарушения обмена веществ в организме матери

D.*Наследственные нарушения в иммунной системе

E.Нарушения в процессе эмбрионального развития

56.У больного обнаружена аутоимунная гемолитическая анемия, развивающаяся по цитотоксическому типу. Какие вещества являются антигенами при аллергических реакциях II типа?

A.Модуляторы воспаления

B.Модифицированные рецепторы клеточных мембран

C.Гормоны

D.*Сывороточные белки

E.Антибиотики

57.Ребенок родился с волчьей пастью. При обследовании обнаружены пороки аорты, в крови - уменьшение Т-лимфоцитов. Какой иммунодефицитный синдром у новорожденного?

A.*Ди Джорджи

B.Швейцарский тип

C.ЛуиБарр

D.ВискоттаОлдрича

E.ЧедиакаХигаси

58.У больного после введения новокаина развился анафилактический шок. Какие антитела ответственны за развитие этой аллергической реакции?

A.*Ig E

B.Ig А

C.Ig D

D.Ig М

E.Ig G

59.При аллергических реакциях немедленного типа возникает дегрануляция тканевых базофилов, которые выделяют биологически активные вещества. Одним из таких веществ является:

A.*Гистамин

B.Ацетилхолин

C.Профибринолизин

D.Фактор Хагемана

E.Система комплемента

60.Больному произведена трансплантация почки. Через несколько суток наступило отторжение трансплантата. К какому типу аллергических реакций относится это осложнение?

A.*Замедленного типа

B.Немедленного типа

C.Анафилаксии

D.Атопии

E.-

61.Профилактическая вакцинация ослабленными микроорганизмами вызывает в организме выработку антител к этим микробам. Какие клетки относятся к антителопродуцирующим клеткам иммунной системы?

A.*Плазмоциты

B.Т-лимфоциты

C.Макрофаги

D.NK-клетки

E.Т-хелперы

62.У ребенка через 1 час после употребления поливитаминов в виде сиропа появились высыпания по всему телу по типу крапивницы с выраженным ощущением зуда. К какому типу аллергической реакции относятся данные проявления?

A.*Анафилактическому

B.Имунокомплексному

C.Цитотоксическому

D.Гиперчувствительности замедленного типа

E.Аутоалергическому

63.У ребенка после того как он съел клубнику, появились зудящие красные пятна на коже (крапивница). К какому типу аллергических реакций по классификации Джелла и Кумбса относится эта реакция?

A.*Реагиновому (анафилактического)

B.Цитотоксическому (цитолиза)

C.Иммунокомплексному (реакции феномена Артюса)

D.Клеточно-опосредствованному

E.Стимулирующему

64.У ребенка после употребления клубники появились зудящие красные пятна на коже, то есть возникла крапивниця. С каким компонентом иммунной системы взаимодействовал аллерген в организме?

A.*IgE.

B.IGM.

C.IGA.

D.Т-хелперами

E.Т-еффекторами

65.У девочки после приема в пищу клубники, появились зудящие красные пятна на коже (крапивница). Какое из биологически активных веществ выделяется при дегрануляции тканевых базофилов?

A.*Гистамин

B.Брадикинин

C.Простагландин И2

D.Белки комплемента

E.Интерлейкин-1

66.У больного П. после травмы возникла необходимость введения противостолбнячной сыворотки, однако проба на чувствительность к сыворотке оказалась положительной. Как провести специфическую гипосенсибилизацию у больного? Введением:

A.*Малых дробных доз специфического аллергена

B.Физиологических доз глюкокортикоидов

C.Разрешающей дозы специфического аллергена

D.Лечебных доз антигистаминных препаратов

E.Наркотических веществ, снижающих чувствительность

67.У ребенка двух лет установлен диагноз гипоплазии тимуса. Какой показатель состояния иммунной системы является наиболее характерным для этого иммунодефицита?

A.*Снижение количества Т-лимфоцитов

B.Снижение количества В-лимфоцитов

C.Дефицит Т и В-лимфоцитов

D.Отсутствие плазматических клеток

E.Снижение иммуноглобулинов М

68.У пациента с бронхиальной астмой с помощью кожных аллергических проб

установлена сенсибилизация аллергеном тополиного пуха. Какой фактор иммунной системы играет решающую роль в развитии этого иммунопатологического состояния?

A.*IGE.

B.IGD.

C.IGM.

D.Сенсибилизированные Т-лимфоциты

69.Во время проведения хирургических манипуляций был использован новокаин с целью обезболивания. Через 10 мин. у больного появились бледность кожных покровов, одышка гипотензия. Аллергическую реакцию какого типа можно заподозрить?

A.*Анафилактического

B.Цитотоксического

C.Иммунокомплексного

D.Стимулирующего

E.Клеточно-опосредствованного

70.Больному с целью обезболивания, ввели раствор местного анестетика. Через несколько минут у больного развилась одышка, тахикардия, потеря сознания. Какой шок развился у больного?

A.*Анафилактический

B.Кардиогенный

C.Геморрагический

D.Травматический

E.Ожоговый

71.У госпіталізованого пацієнта встановлена недостатність імунітету, яка призвела до зниження резистентності до вірусної інфекції. Дефіцит яких клітин є найбільш вірогідним?

A.*Т-лімфоцитів

B.В-лімфоцитів

C.Макрофагів

D.Нейтрофілів

E.Фібробластів

72.У больного с явлениями аллергии отмечаются нарушение носового дыхания, серозные выделения из носа, покраснение глаз и слезотечение. Какой стадии аллергической реакции соответствуют указанные проявления?

A.Биохимической

B.*Патофизиологической

C.Иммунологической

D.Пролиферации

E.Компенсации

73.У хворого після введення парентерально вітаміну В6 розвинувся анафілактичний шок з явищами бронхоспазму, зниженням АТ, ціанозом та судомами. Який медіатор анафілаксії спричинює падіння артеріального тиску?

A.Гепарин

B.*Гістамін

C.Катехоламіни

D.Глюкокортикоїди

E.Тромбоксан

74. Після ін”єкції препарату у хворого виникли свербіння, висипання на шкірі, утруднене дихання, артеріальний тиск складає 70/40 мм.рт.ст., запаморочення. Яка алергічна реакція

за Кумбсом і Джелом імовірно розвинулася у хворого?

A.Цитолізу

B.*Анафілактична

C.Типу феномена Артюса

D.Сповільненого типу

E.Стимулюючого типу

75.На якій стадії алергічної реакції відбувається виділення та утворення медіаторів алергії?

A.Прогресії

B.Патофізіологічна

C.Імунологічна

D.Ексудації

E.*Патохімічна

76.Через декілька годин після введення туберкуліну в місці його введення виникли почервоніння, свербіж, ущільнення шкіри. Розмір ділянки запалення та інтенсивність його проявів збільшувалися впродовж 48 годин. До якого типу алергічних реакцій належить позитивна туберкулінова проба?

A.Стимулюючого.

B.Анафілактичного.

C.Цитотоксичного.

D.Імунокомплексного.

E.*Гіперчутливості сповільненого типу.

77.В стоматологічному кабінеті районної поліклініки після введення лідокаїну у хворого розвинувся анафілактичний шок. Які з перелічених імуноглобулинів зумовлюють розвиток анафілактичного шоку?

A.*Ig E

B.Ig M

C.Ig A

D.-

E.Ig Д

78.У больного, получившего травму в автомобильной катастрофе, через 7 дней появились первые симптомы столбняка. Ему назначили курс лечения противостолбнячной сыворотки и больной начал выздоравливать. Через две недели у больного повысилась температура, увеличились лимфоузлы, появилась отечность суставов, сыпь, зуд и нарушение со стороны сердечно-сосудистой системы. Как называется состояние, которое возникло у больного?

A.*Сывороточная болезнь

B.Крапивница

C.Анафилактический шок

D.Дисбактериоз

E.Отек Квинке

79. У хворих на бруцельоз спостерігається позитивна шкірна проба Бюрне. Який фактор імунної системи відіграє вирішальну роль у розвитку запальної реакції в місці введення бруцеліну цим пацієнтам?

A.*Сенсибілізовані Т-лімфоцити.

B.IgA.

C.IgE.

D.IgG. E. IgD.

80.Для діагностики генералізованої герпетичної інфекції досліджено сироватку крові з метою вивлення специфічних антитіл певного класу. Антитіла якого класу свідчать про гостру стадію вірусної інфекції?

A.*Ig M

B.Ig A

C.Ig E

D.Ig G

E.Ig D

81.З метою діагностики ВІЛ-інфекції досліджують сироватку крові для виявлення специфічних антитіл методом твердофазного імуноферментного аналізу. Які ензиммічені антитіла при цьому використовують?

A.*Проти імуноглобулінів людини.

B.Проти антигенів ВІЛ.

C.Проти білка gp120.

D.Проти білка gp17.

E.Проти білка gp41.

82.Жінка Д., 54-ти років звернулася до лікаря зі скаргами на непереносимість курячих яєць, що з’явилася нещодавно. Антигістамінні препарати, що їх призначив лікар призводили до деякого покращення стану хворої. Які антитіла могли сприяти розвитку цієї реакції?

A.*Ig E

B.Ig А

C.Ig D

D.Ig G

E.Ig M

83. Для моделювання анафілактичного шоку у морської свинки провели пасивну сенсибілізацію. Що слід ввести з метою пасивної сенсибілізації?

A.*Специфічні імуноглобуліни.

B.Кінську сироватку

C.Сенсибілізовані Т- лімфоцити

D.Тканинні базофіли

E.В-лімфоцити

84.При розвитку анафілактичних реакцій спостерегаються виражені гіперемія, набряк, біль. Який медіатор анафілаксії визначає розвиток вищезгаданих розладів?

A.*Гістамін

B.Гепарин

C.Фактори хемотаксису

D.Фактор активації тромбоцитів

E.Білки комплементу

85.Відразу після повторного введення антибіотіка у пацієнта з'явилися задуха, відчуття страху, зниження артеріального тиску. Алергичнаі реакція якого типу лежать в основі цього стану у хворого?

A.*Анафілактичні

B.Гуморальні цитотоксичні

C.Імунокомплексні

D.Клітинні цитотоксичні

E.Стимулюючі

86.У піцієнтки 23-х років виявлено, що після використання нової губної помади з’явилися набряк і свербіння губ, а через 2 дні – кірочки на червоній облямівці губ. Який тип алергіїчної реакції найбільш вірогідний?

A.*Анафілактична

B.Цитотоксична

C.Імунокомплексна

D.Сповільнена

E.Стимулююча

87.Через декілька хвилин після внутрішньовенного введення препарату артеріальний тиск у пацієнта знизився до 70/30 мм. рт. ст. Який з хімічних медіаторів анафілаксії спричинює вазоділатацію і шок?

A.*Гістамін

B.Гепарин

C.Інтерлейкіни

D.Фактор хемотаксису еозинофілів

E.Фактор хемотаксису нейтрофілів

88.При обстеженні хворого, який тривалий час приймає глюкокортикоїди, виявлена лімфопенія. Як можна охарактеризувати функціональний стан імунної системи пацієнта?

A.Імунодефіцит вторинний

B.Імунодефіцит первинний

C.Імунодефіцит вроджений

D.Толерантність до ауто антигенів

E.Анафілаксія

89.У жінки, яка відпочивала на дачі, відразу після укусу оси виник біль, через кілька хвилин на шкірі в місці укусу з’явився пухир, еритема і сильне свербіння, а ще через деякий час - кропив'янка, експіраторна задишка. Внаслідок дії яких факторів у хворої розвинулась експіраторна задишка?

A.* Гістаміну

B.Фактора Хагемана

C.Лізосомальних ферментів

D.Норадреналіну

E.Адреналіну

90.У чоловіка в кінці весни з’являються ознаки реніту, почервоніння кон’юктиви очей. В крові виявлено підвищений зміст еозінофілів. Який тип алергічної рекції у чоловіка?

A.*Анафілактичний.

B.Цитотоксичний.

C.Імунокомплексний.

D.Гіперчутливість сповільненого типу.

E.Стимулюючий.

91.При повторном введении аллергена у морской свинки начинается выделение гистамина тучными клетками крови. К какому уровню реактивности относится такой ответ организма?

A.* Клеточному

B.Субклеточному

C.Молекулярному

D.Органному

E.Системному

92.У больного после введения противостолбнячной сыворотки развился анафилактический шок. Какие клетки выделяют медиаторы при классическом варианте протекания анафилаксии?

A.*Тучные клетки

B.Эозинофилы

C.Т-лимфоциты

D.Нейтрофилы

E.В-лимфоциты

93.Лікарем швидкої допомоги у пацієнта був діагностований анафілактичний шок, що супроводжувався бронхоспазмом. Виділення якої біологічно активної речовини тканинними базофілами зумовлює таку клінічну симптоматику?

A.*Гістаміну

B.Гепарину

C.Брадикініну

D.Простагландинів

E.Лейкотрієнів

94.Під час сінокосу в одного з робітників піднялась температура тіла, з’явився озноб, сльозотеча, нежить. Робітник сказав, що у нього це спостерігається щорічно в таку пору. Який тип алергічної реакції за Кумбсом і Джеллом розвинувся у робітника?

A.* Тип I.

B.Тип II.

C.Тип III.

D.Тип IV.

E.Тип V.

95.При обследовании у аллерголога больному поставлен диагноз – полиноз. Каким способом можно провести специфическую десенсибилизацию?

A.*Дробным введением аллергена

B.Антигистаминными препаратами

C.Глюкокортикоидами

D.Введением физиологического раствора

E.-

96.У жінки, якій перед видаленням зуба зробили ін’єкцію анестетика, раптово почервоніло обличчя, з’явилась задишка, почали набрякати губи. Дія яких факторів зумовлює розвиток набряку в даній ситуації?

A.* Гістаміну

B.Імуноглобулінів

C.Фактора Хагемана

D.Лізосомальних ферментів

E.Адреналіну

97. Чоловіку,40-ти років,при лікуванні пульпіту був введений розчин лідокаїну.Через декілька хвилин у хворого розвинулась тахікардія,різке зниження артеріального тиску. Який з нижчеперечисленних станів розвинувся у хворого?

A.*Анафілактичний шок.

B.Краш-синдром.

C.Бронхіальний спазм.

D.Стрес-адапційний синдром.

E.Опіковий шок.

98.Мальчик на втором году жизни стал часто болеть респираторными заболеваниями, стоматитами, гнойничковыми поражениями кожи. Даже небольшие повреждения десен и слизистой осложняются длительно протекающим воспалением. Установлено, что в крови ребенка практически отсутствуют иммуноглобулины всех классов. Снижение функциональной активности какой клеточной популяции лежит в основе описанного синдрома?

A.*В-лимфоцитов

B.Т-лимфоцитов

C.Нейтрофилов

D.Макрофагов

E.NK-лимфоцитов

99.У пациента, после лечения у стоматолога, появились зудящие красные пятна на коже. Был поставлен диагноз крапивница. Какой компонент иммунной системы вступает во взаимодействие с аллергеном при этом типе аллергической реакции?

A.*IgЕ

B.Т-хелперами.

C.Т-эффекторами

D.Ig.А

E.IgМ

100.У пациента, через 30 минут после лечения у стоматолога, появились красные зудящие пятна на коже лица и слизистой рта. Был поставлен диагноз крапивница. Какое из биологически активных веществ, вызывающее расширение сосудов, появление зуда, выделяется при этом типе аллергической реакции?

A.*Гистамин

B.Простагландин Е2

C.Лейкотриен В4

D.Интерлейкин-1

E.Брадикинин

101.Пациенту перед экстракцией зуба была проведена проводниковая анестезия новокаином, после введения которого появились отек и гиперемия вокруг места укола, зуд кожи, общая слабость, гипотензия, двигательное возбуждение. Определите, как называется возникшее осложнение?

A.*аллергия

B.идиосинкразия

C.тахифилаксия

D.лекарственная зависимость

E.воспаление

102.В приймальне відділення інфекційної лікарні поступив чоловік 25 р. після обстеження був встановлений діагноз СНІД. Ураження яких клітин обумовлює стан імунодефіциту?

A.*Т-хелпери

B.Т-кіллери

C.Т-супресори

D.Плазмоцити

E.Тучні клітини (ткан. базофіли)

103.У юноши 20-ти лет травмированное правое яичко. Какую опасность это может составлять для левого (здорового) яичка на 2-3 неделе после травмы?

А. Развитие атрофии

B.Развитие инфекционного процесса

C.*Демаскирование антигена и возникновение повреждения антителами

D.Развитие гипертрофии

E.Не угрожает ничем

104. У мальчика 5-ти месяцев при исследовании иммунного статуса выявлено уменьшение имуноглобулинов, особенно IgA и IgM. В крови и лимфатических узлах отсутствуют В-лимоциты и плазматические клетки. Реакции Т-лимфоцитов сохранены. Заболевание передается по наследственности как сцепленное с полом. Какая патология наблюдается у этого ребенка?

А.* Болезнь Брутона

B.Синдром Луи-Барр

C.Синдром Вискотта-Олдрича

D.Иммунодефицит швейцарского типа

E.Ранняя гипогаммаглобулинемия

105.У ВИЧ-инфицированного больного наблюдается угнетение активности иммунной системы. Поражение каких клеток в наибольшей степени обусловливает состояние иммунодефицита у этого больного?

А. Т-киллеров

B.Т-супресоров

C.Макрофагов

D.В-лимфоцитов

E.*Т-хелперов

106.При исследовании состояния иммунной системы больного с хроническими грибковыми поражениями кожи выявлено нарушение клеточного иммунитета. Снижение каких показателей наиболее характерно для этого?

А. Плазмоцитов

B.Иммуноглобулинов G

C.Иммуноглобулинов E

D.В-лимфоцитов

E.*Т-лимфоцитов

107.Мальчик 1,5 лет постоянно болеет пиодермией и трижды болел пневмонией. В крови снижено количество имуноглобулинов G и А. Какой вид иммунодефицита возник у ребенка?

А. Швейцарский тип

B.*Гипогаммаглобулинемия Брутона

C.Гипоплазия вилочковой железы

D.Синдром Вискотта -Олдрича

E.Синдром Луи-Барр

108. Японский доктор Масуги вызывал развитие гломерулонефрита у крыс введением гомогената почек крысы кролику. Через несколько недель сыворотку сенсибилизованого

кролика вводили крысам. Какой тип аллергической реакции по Джеллу и Кумбсу лежит в основе развития гломерулонефрита у крыс?

А.Анафилактический B. *Цитотоксический C. Иммунокомплексный

D. Гиперчувствительность замедленного типа E. Стимулирующий

109.С целью снижения аутоиммунных реакций после пересадки органов обязательным является проведение курса гормонотерапии. Какие гормоны применяют с этой целью?

А.Адреналин.

B.Минералокортикоиды.

C.Половые гормоны.

D.*Глюкокортикоиды.

E.Соматотропный гормон.

110.Предварительно сенсибилизированной морской свинке внутривенно ввели 10 мл лошадиной сыворотки и через полминуты отмечено: шерсть взъерошена, животное чихает, кашляет, почёсывает мордочку, дыхание затруднённое, судорожное, непроизвольная дефекация и мочеиспускание. Какая стадия анафилактического шока наблюдается у животного?

A.Патохимическая

B.Сенсибилизации

C.*Патофизиологическая

D.Иммунологическая

E.-

111.После перенесенной стрептококковой инфекции у мужчины диагностирован острый гломеролонефрит. Наиболее вероятно, что поражение базальной мембраны клубочков связано с аллергической реакцией. Какого типа?

А. Замедленного типа

B.Анафилактического типа

C.Цитотоксичного типа

D.*Иммунокомплексного типа

E.Стимулирующего типа

112.Ребенок во время игры порезал ногу осколком стекла и был направлен в поликлинику для введения противостолбнячной сыворотки. С целью предупреждения развития анафилактического шока лечебную сыворотку вводили по Безредке. Какой механизм лежит в основе подобного способа гипосенсибилизации организма?

A.Стимуляция синтеза антиген-специфичных IgG

B.Блокирование синтеза медиаторов в тучных клетках

C.Стимуляция иммунологической толерантности к антигену

D.*Связывание фиксированных на тучных клетках IgE

E.Связывание рецепторов к IgE на тучных клетках

113.У хворого діагностовано тиротоксикоз. У крові знайдено антитиреоїдні антитіла. Який тип алергічної реакції за Кумбсом і Джелом спостерігається при розвитку цього захворювання?

A.Цитотоксичний

B.Анафілактичний

C.*Стимулюючий

D.Імунокомплексний

E.Гіперчутливість сповільненого типу

114.Жінка 27-ми років звернулась зі скаргами на свербіння та печію в очах, сльозотечію, чхання, виділення з носу. Симптоми з'явились після поїздки за місто влітку. Діагностовано поліноз. Якого типу алергічна реакція розвилась при наявному захворюванні?

A.Стимулююча

B.Цитотоксична

C.Реакція утворення імунних комплексів

D.Сповільненої чутливості

E.*Анафілактична

115.У дівчинки 14-ти років, експіраторна задишка. Стан розвився після гри з собакою. В анамнезі хворіє бронхіальною астмою. Якою із перелічених нижче біологічно активною речовиною найбільш вірогідно викликаний спазм гладеньких м'язів бронхіол дівчинки?

A.Серотонін

B.Тромбоксан А2

C.*Лейкотрієн Д4

D.Брадикінін

E.Ацетілхолін

116.У ребенка 6-ти лет, часто болеющего респираторными заболеваниями, отмечаются экзематозные явления после приема цитрусовых, склонность к затяжному течению воспалительных процессов. Какой вид диатеза можно предположить в данном случае?

A.Астенический

B.Геморрагический

C.Лимфатико-гипопластический

D.Нервно-артритический

E.*Экссудативно-катаральный

117.У хворого А. в серпні після праці на дачі развився стан, що характеризувався лікарем, як стан підвищенної і якісно зміненої реакції на надходження до організму сполук антигенної або гаптенної природи. Який з перерахованих станів найбільш підходить під описану лікарем характеристику?

A.Параалергія

B.Анафілаксія

C.*Алергія

D.Тахіфілаксія

E.Імунологічна толерантність

118.Через 5-8 дней после применения значительных количеств лечебной сыворотки у больного выявились кожные высыпания, зуд, припухлость, боли в суставах, повысилась температура тела, в моче появился белок. Был поставлен диагноз сывороточная болезнь. Что является важным фактором в патогенезе этого синдрома?

A.Активация Т-киллеров.

B.Дегрануляция тканевых базофилов.

C.*Накопление в крови циркулирующих иммунных комплексов

D.Активация макрофагов

E.Цитолиз форменных элементов крови

119.Після прийому амідопірину у хворого виникла лейкопенія. В крові знайдені

антилейкоцитарні антитіла. Який тип алергічної реакції за Кумбсом і Джеллом виник у даному випадку?

A.Імунокомплексний.

B.Стимулюючий

C.Анафілактичний.

D.Гіперчутливість сповільненого типу.

E.*Цитотоксичний.

120.У дитини двох років встановлено діагноз гіпоплазії тимуса. Який показник стану імунної системи є найбільш характерним для цього імунодефіцита?

A.Дефіцит Т и В-лімфоцитів

B.Зниження кількості В-лімфоцитів

C.*Зниження кількості Т-лімфоцитів

D.Відсутність плазматичних клітин

E.Зниження іммуноглобулінів М

121.У девушки 18-ти лет через 5 часов после приёма морепродуктов на коже туловища и дистальных отделов конечностей появились маленькие зудящие папулы, частью сливающиеся между собой. Через сутки высыпания самопроизвольно исчезли. Назовите механизм гиперчувствительности, лежащий в основе данных изменений:

A.Иммунокомплексная гиперчувствительность

B.*Атопия (местная анафилаксия)

C.Системная анафилаксия

D.Антителоопосредованный клеточный цитолиз

E.Клеточная цитотоксичность

122.У больного переливание крови осложнилось развитием гемотрансфузионного шока. Назовите тип аллергической реакции, лежащей в основе данной патологии:

A.Иммунокомплексный

B.Рецепторноопосредованный

C.Гиперчувствительность замедленного типа

D.*Цитотоксический

E.Анафилактический

123.Какое состояние может развиться через 15-30 минут после повторного введения антигена вследствие повышенного уровня антител, преимущественно IgE, которые адсорбируются на поверхности клеток мишеней – тканевых базофилов (тучных клеток) и базофилов крови?

A.*Анафилаксия

B.Иммунокомплексная гиперчувствительность

C.Сывороточная болезнь

D.Гиперчувствительность замедленного типа

E.Антитело-зависимая цитотоксичность

124.У больных с синдромом приобретённого иммунодефицита (СПИД) резко снижается иммунологическая реактивность, что проявляется развитием хронических воспалительных процессов, инфекционных заболеваний, опухолевого роста. Клетки какого типа повреждает ВИЧ-инфекция, вследствие чего снижается иммунная защита?

A.В-лимфоциты

B.Т-супрессоры

C.*Т4-хелперы

D.Естественные киллеры (NK)

E.Е8-эффекторы

125.У ребенка 10-ти лет через 2 недели после перенесенной ангины развился нефротический синдром (протеинурия, гематурия, цилиндрурия), что свидетельствует о поражении базальной мембраны клубочков почек. Какой наиболее вероятный механизм лежит в основе повреждения базальной мембраны?

A.*Иммунокомплексный

B.Грунулематозный

C.Антительный

D.Цитотоксический

E.Реагиновый

126.У мужчины 36-ти лет после перенесенной стрептококковой инфекции диагностирован острый гломерулонефрит. Наиболее вероятно, что поражение базальной мембраны почечных телец возникает вследствие аллергической реакции такого типа:

A.*Иммунокомплексная

B.Замедленная

C.Цитотоксическая

D.Стимулирующая

E.Анафилактическая

127.Мужчине для диагностики туберкулёза был введен туберкулин,. На месте введения возникло покраснение, которое в течении двух суток увеличивалось в размерах. Какой тип аллергической реакции развился у человека на туберкулин?

A.Анафилактический

B.

C.*Клеточно-опосредованный

D.Иммунокомплексный

E.Стимулирующая аллергическая реакция

128.У відповідь на застосування знеболюючого засобу при екстракції зуба у хворого з’явились: виражений набряк м'яких тканин нижньої та верхньої щелеп, висип на шкірі обличчя, почервоніння, свербіж. Який з патологічних процесів лежить в основі такої реакції на анестетик?

A.* Алергія.

B.Токсична дія препарату.

C.Запалення.

D.Недостатність кровообігу.

E.Порушення лімфовідтоку.

129.Больному для обезболивания при удалении кариозного зуба врачомстоматологом был введен раствор новокаина. Через несколько минут у больного появились симптомы: падение АД, учащение дыхания, потеря сознания, судороги. К какому типу аллергических реакций можно отнести это состояние?

A.*Анафилактическому

B.Цитотоксическому

C.Иммунокомплексному

D.Замедленной гиперчувствительности

E.Стимулирующему

130.Хірург-стоматолог перед екстракцією зуба з метою знеболення ввів пацієнту ультракаїн. Проба на чутливість проведена не була. Через декілька хвилин після введення

препарату у хворого розвинувся анафілактичний шок. Які клітини виробляють реагіни, що беруть участь в реакціях анафілактичного типу?

A.*Плазматичні клітини.

B.В-лімфоцити.

C.Т-лімфоцити.

D.Тучні клітини.

E.Еозинофіли.

131.У підлітка було видалено зуб з застосуванням новокаїну. Через 10 хвилин у нього з явились блідість шкірних покривів, задишка, розвилася гіпотензія. Який тип алергічної реакції виник у підлітка?

A.*Анафілактична

B.Цитотоксична

C.Імунокомплексна

D.Клітинно-опосредкована

E.Стимулююча

132.У підлітка було видалено зуба з застосуванням новокаїну. Через 10 хвили у нього появились блідність шкіряних покривів, задишка, розвилась гіпотензія. З чим реагує на тканинних базофілах алерген?

A.*IgE.

B.IgА.

C.IgД.

D.ІgМ.

E.Т-лімфоцитами

133.Ліквідатору наслідків аварії на АЕС, що отримав велику дозу опромінення, проведено трансплантацію кісткового мозку. Через деякий час після проведеної операції у пацієнта діагностовано розвиток реакції трансплантат проти хазяїна. Які антигени послужили пусковим механізмом виникнення цієї реакції?

A.*Антигени системи HLA клітин організму ліквідатора.

B.Антигени системи Rh еритроцитів ліквідатора.

C.Антигени HBs, HBc, HBE.

D.Антигенами системи ABO еритроцитів ліквідатора.

E.Антигени системи HLA клітин організму донора.

134.У хворого стоматологічної лікарні запідозрено ВІЛ-інфекцію. Які з перелічених субпопуляцій імунокомпетентних клітин пошкоджується при СНІД в першу чергу?

A.Т-хелпери

B.Т-супресори

C.Т-кілери

D.В-лімфоцити

E.Макрофаги

ЭКСПЕРИМЕНТ КАК ОСНОВНОЙ МЕТОД ПАТОФИЗИОЛОГИЧЕСКОГО ИССЛЕДОВАНИЯ.

1.Животному введено пирогенное вещество. После этого на протяжении суток у него изучали взаимоотношения между степенью повышения температуры тела и частотой пульса. Какой метод изучения был использован в данном случае?

A.Наблюдения

B.Острого эксперимента

C.*Хронического эксперимента

2.Животному, у которого вызвана лихорадка, ввели жаропонижающее вещество и наблюдают за тем, как у него после этого изменяется температура тела. Какой этап патофизиологического эксперимента проводится в данном случае?

A.Определение исходных показателей

B.Моделирование патологического процесса

C.Изучение динамики развития и течения процесса

D.*Экспериментальная терапия

E.Планирование

3.Кролику в гипоталамическую область вживлены электроды. На протяжении нескольких недель у него изучают по электрофизиологическим показателям функциональную активность гипоталамуса при хроническом воспалении. Как правильно называется такой вид эксперимента?

A.Острый

B.Вивисекция

C.Физиологический

D.*Хронический

E.Биологический

4.Исследователю в эксперименте на животном нужно изучить, как изменяются функции сердечно-сосудистой системы при лихорадке. Какое из перечисленных животных нельзя использовать для проведения этого эксперимента?

A.*Лягушку

B.Кролика

C.Белую крысу

D.Собаку

E.Морскую свинку

5.Исследователю необходимо изучить влияние острой кровопотери на состояние сердечной деятельности и артериального давления в эксперименте. Какой вид эксперимента будет использован для данного исследования?

A.Биологический

B.Физиологический

C.*Острый

D.Хронический

E.Патохимический

6.Исследователю необходимо изучить влияние кровопотери на изменение объема циркулирующей крови в постгеморрагическом периоде. С этой целью была избрана методика гемодилюции. Каким методом эксперимента следует воспользоваться в данном случае?

A.Острым

B.Вивисекции

C.Физиологическим

D.Биологическим

E.*Хроническим

7.Экспериментатор собирается, используя методику полярографического определения напряжения кислорода в мышечной ткани, изучить состояние этого показателя на разных этапах развития лихорадочной реакции у животного. Какой вид эксперимента наиболее выгодно использовать в данном случае?

A.Вивисекцию

B.Физиологический

C.*Хронический

D.Биологический

E.Острый

8.Классическим примером изучения роли нервной системы в углеводном обмене является «сахарный укол» К.Бернара. Какой метод исследования был использован в данном случае?

A.Хронического исследования

B.*Вивисекции

C.Наблюдения

D.Моделирования

E.Статистический

9.У животного в эксперименте был вызван тиреотоксикоз. Для подтверждения этого состояния необходимо определить уровень обмена веществ в зависимости от приема пищи и физической нагрузки. Каким экспериментальным методом следует воспользоваться в данном случае?

A.Определения содержания йода в щитовидной железе

B.*Прямой калориметрии

C.Определения концентрации органического йода в крови

D.Определения напряжения кислорода в крови

E.Определения содержания гормонов щитовидной железы в крови

10.К.Бернар в 1851 році детально вивчив прояви нейропаралітичної артеріальної гіперемії на вусі кроля. Яка експериментальна методика була ним використана?

A.*Методика вилучення

B.Методика подразнення

C.Методика введення лікарських речовин

D.Методика ізольованого органу

E.Методика перевантаження

Наследственность.

1.Наследственные заболевания могут быть связаны с нарушениями структуры и количества хромосом или генов. Какие из перечисленных заболеваний относятся к моногенным заболеваниям?

A.*Гемофилия

B.Сахарный диабет 1 типа

C.Синдром Шерешевского

D.Синдром Клайнфельтера

E.Болезнь Дауна

2.При проведенні амніоцентезу в клітинах плода виявлено по 2 тільця статевого хроматину (тільця Барра). Для якого захворювання характерна дана ознака?

A.*Трисомія X

B.Синдром Клайнфельтера

C.Синдром Шерешевського-Тернера

D.Синдром Дауна

E.Синдром Патау

3.В медико-генетической консультации при обследовании больного мальчика в крови были выявлены нейтрофильные лейкоциты с 1 "барабанной палочкой". Наличие какого синдрома возможно у мальчика?

А. Синдром Дауна

B.*Синдром Клайнфельтера.

C.Синдром Шерешевского-Тернера

D.Синдром Едвардса

E.Синдром трисомии - Х

4.У молодого человека в возрасте 20-ти лет, высокого роста и астенического телосложения с признаками гипогонадизма, гинекомастией и уменьшенной продукцией спермы [азооспермия] выявлен кариотип 47 ХХY. Какой наследственный синдром сопровождается такой хромосомной аномалией?

А. Дауна

B.Вискотта-Олдрича

C.Тернера

D.Луи-Барра

E.*Клайнфельтера

5.В медико-генетическую консультацию за рекомендацией андролога обратился мужчина

35лет по поводу отклонений физического и психического развития. Объективно установлено: высокий рост, астеническое телосложение, гинекомастия, умственная отсталость. При микроскопии клеток слизистой оболочки ротовой полости найден в 30% половой хроматин (одно тельце Барра). Каков наиболее достоверный диагноз?

А. Болезнь Дауна

B.Синдром Ди Джордже.

C.*Синдром Клайнфельтера.

D.Болезнь Реклингаузена.

E.Болезнь Иценко-Кушинга.

6.Возникновение нижеперечисленных заболеваний связано с генетическими

факторами. Назовите патологию с наследственной предрасположенностью. А. Дальтонизм

B.Хорея Гентингтона

C.Фенилкетонурия

D.Серповидноклеточная анемия

E.*Сахарный диабет

7.При обследовании буккального эпителия мужчины был обнаружен половой хроматин. Для какой хромосомной болезни это характерно?

A.Болезнь Дауна

B.*Синдром Клайнфельтера

C.Синдром Шерешевского-Тернера

D.Трисомия по Х-хромосоме

E.Гипофосфатемический рахит

8.У букальних мазках епітелію жінки виявлено в ядрі клітини 2 тільця Барра. Це характерно для синдрому:

A.*Трисомія статевих хромосом

B.Трисомія 21-ї хромосоми

C.Трисомія 13-ї хромосоми

D.Трисомія по У-хромосомі

E.Моносомія статевих хромосом

9.Кровотечу, що виникла у дитини після видалення зуба, не вдавалося припинити протягом 6 годин. Проведене дослідження системи гемостазу встановило різке зменшення вмісту VIII фактору зсідання крові. За яким типом спадкується це захворювання?

A.*Зчеплене із статевою хромосомою

B.Аутосомно-домінантний

C.Аутосомно-рецисивний

D.Полігенний

E.Неповне домінування

10.У хлопчика 10 років з розумовою відсталістю характерний зовнішній вигляд: невелика голова з скошеною потилицею, косі очні лунки, короткий ніс з широкою переносицею, напіввідкритий рот. Наглядається у лікаря в зв’язку з уродженим пороком серця. Яким порушенням каріотипу обумовлена патологія у хлопчика?

A.*Трисомія по 21-ій парі хромосом.

B.Трисомія по 13-ій парі хромосом.

C.Трисомія по 18-ій парі хромосом.

D.Трисомія Х.

E.Моносомія по Х-хромосомі.

11.Чоловік 25 років звернувся по причині безпліддя. Має високий зріст, астенічна будова тіла, відмічається зниження інтелекту. В зіскобі слизової оболонки щоки знайдені тільця Барра. Про яку патологію можна думати?

A.*Синдром Клаєнфельтера.

B.Адіпозоногенітальна дистрофія.

C.Акромегалія.

D.Адреногенітальний синдром.

E.Євнухоїдизм.

РАССТРОЙСТВА ПЕРИФЕРИЧЕСКОГО КРОВООБРАЩЕНИЯ. ТРОМБОЗ. ЭМОЛИЯ.

1.Спортсмен после прыжка приземлился на внешний край стопы. Возникла острая боль в голеностопном суставе, активные движения в нем ограничены, пассивные – в полном объеме, но болезненны. Потом возникла припухлость в области внешней косточки, кожа покраснела, стала теплой наощупь. Какой вид расстройства периферического кровообращения возник в данном случае?

A.Тромбоз

B.*Артериальная гиперемия

C.Стаз

D.Венозная гиперемия

E.Эмболия

2.Женщина 50-ти лет находится на лечении по поводу тупой травмы грудной клетки. На нижних конечностях значительное варикозное расширение вен. Повышение выработки какого вещества будет более всего влиять на возникновение тромбов в данном случае?

A.Простациклина

B.Простагландина F2

C.Лейкотриена C4

D.Простагландина E2

E.*Тромбоксана A2

3.Известно, что в венах тромбы образуются в пять раз чаще, чем в артериях, а в венах нижних конечностей – в три раза чаще, чем в венах верхних конечностей. Укажите, с каким патогенетическим фактором, обусловливающим процесс тромбообразования, связано это явление?

A.Повреждением сосудистой стенки

B.Повышением активности свертывающей системы крови

C.Повышением вязкости крови

D.*Замедлением кровотока в венах

E.Снижением активности противосвертывающей системы крови

4.Ряд патологических процессов (атеросклероз сосудов, воспаление, механическая травма, некроз тканей) сопровождается тромбообразованием. Какой патогенный фактор в механизме тромбообразования играет при этих процессах ведущую роль?

A.*Степень повреждения эндотелия сосудов

B.Замедление кровотока

C.Изменение физико-химических свойств крови

D.Повышение активности свертывающей системы крови

E.Снижение активности противосвертывающей системы крови

5.У больного полицитемией увеличено число эритроцитов в единице объема крови, при этом отмечается склонность к тромбообразованию. С изменением какого показателя физико-химических свойств крови связано это явление?

A.рН

B.Осмотической резистентности эритроцитов

C.Тромбина

D.Фибриногена

E.*Вязкости

6.У больной болезнью Вольденстрема в крови появились необычные белки с очень большой молекулярной массой (макроглобулины). Изменение какой физико-химической особенности крови лежит в основе развития тромбоза сосудов при этом?

A.Нарушение рН

B.*Повышение вязкости крови

C.Повышение активности свертывающей системы крови

D.Снижение активности противосвертывающей системы крови

E.Повышение осмотической резистентности эритроцитов

7.При микроскопии препарата брыжейки лягушки в одной из вен, находящейся вблизи кристаллика соли, на внутренней ее стенке образовался бесцветный конгломерат из форменных элементов. Кровь, протекающая по сосуду в этом месте, обтекает его. Какой вид тромба по локализации и его морфологии образовался в сосуде?

A.*Белый пристеночный

B.Красный пристеночный

C.Белый обтурационный

D.Красный обтурационный

E.Смешанный пристеночный

8.При повреждении иголкой сосудов брыжейки в поле зрения микроскопа наблюдается кровотечение из этого сосуда, которое со временем прекращается, а просвет сосуда в этом месте быстро заполняется сгустком крови, который состоит преимущественно из эритроцитов. Какой вид тромба по локализации и его морфологии образуется в сосуде?

A.Белый пристеночный

B.Красный пристеночный

C.Белый обтурационный

D.*Красный обтурационный

E.Смешанный пристеночный

9.Ранение лодыжки у больной осложнилось возникновением воздушной эмболии. Ранение каких сосудов является причиной этого осложнения?

A.Артерий

B.Артериол

C.Капилляров

D.Венул

E.* Вен

10.При быстром изменении давления воздуха от более высокого к низкому у летчика 50 лет развилась газовая эмболия. Чем обусловлено это явление?

A.Пневмотораксом и деструкцией легких

B.Разрывом крупных вен

C.Тромбообразованием

D.*Взрывной декомпрессией

E.Отрывом атеросклеротических бляшек

11.Наиболее частой формой эндогенных эмболий является тромбоэмболия. Чем объясняется большая частота тромбоэмболии сосудов малого круга кровообращения?

A.Анатомо-функциональной слабостью эндотелия сосудов

B.*Меньшей скоростью кровотока

C.Большей вязкостью крови

D.Большей концентрацией факторов свертывания крови

12.У больной тромбофлебитом возникла эмболия малого круга кровообращения со снижением артериального давления в большом круге кровообращения и развитием острой сердечной недостаточности. Каков основной механизм гипотензии в данном случае?

A.Механическая закупорка легочной артерии, снижение МОК

B.*Рефлекторное снижение АД

C.Гипоксия миокарда вследствие спазма коронарных артерий

D.Аритмии, ведущие к нарушению насосной функции сердца

E.Недостаточность правого желудочка

13.У больного 65-ти лет, страдающего атеросклерозом, диагностирована эмболия артерии правой лодыжки, однако клинические проявления этого осложнения практически отсутствуют. Какой фактор обусловливает это явление?

A.Локализация эмбола

B.Небольшой общий вес эмбола

C.Возраст больного

D.Пол больного

E.*Развитые коллатерали в месте эмболии

14.У больного образовавшийся эмбол большого круга кровообращения снова с током крови попал в сосуды большого круга, минуя малый круг кровообращения. Какая форма патологии сердечно-сосудистой системы является причиной этой эмболии?

A.Большое количество артериоло-венулярных анастомозов

B.*Дефект межпредсердной (либо межжелудочковой) перегородки

C.Деструкция сосудов легких

D.Артериальная гипертензия

E.Сердечная недостаточность

15.У больного с пародонтитом отмечается отек десен. Они имеют темно-красный цвет. Какое местное нарушение кровообращения наблюдается в деснах больного?

A.*Венозная гиперемия

B.Артериальная гиперемия

C.Ишемия

D.Эмболия

E.Тромбоз

16.У пилота на высоте 14000 метров случилась аварийная разгерметизация кабины. Какой вид эмболии у него развился?

A.*Газовая

B.Тромбоэмболия

C.Воздушная

D.Жировая

E.5.Эмболия инородным телом

17.Больной 73-х лет доставлен в больницу с закрытым переломом правой бедренной кости. Внезапно состояние ухудшилось, диагностирована эмболия сосудов головного мозга. Назовите вид эмболии, которая наиболее часто возникает при переломах трубчатых костей:

A.Тканевая

B.*Жировая

C.Газовая

D.Ретроградная

E.Воздушная

18.К врачу обратился больной, который несколько дней назад поранил кисть левой руки. Жалуется на боль в месте повреждения, ограничение движений пальцев. При осмотре установлено, что кисть увеличена в объеме, гиперемирована, температура кожи на ней повышена. О каком патологическом процессе свидетельствуют данные признаки?

A.Тромбозе

B.Опухоли

C.*Воспалении

D.Эмболии

E.Лимфостазе

19.В некоторых случаях патологии человека при образовании эмбола в большом круге кровообращения он может снова с током крови попадать в сосуды большого круга, минуя малый круг. Как называется такой вид жэмболии?

A.Ретроградная

B.Газовая

C.Тканевая

D.Жировая

E.*Парадоксальная

20.У больного с переломом голеностопного сустава после снятия гипсовой повязки наблюдается отек стопы, цианоз, местное понижение температуры, увеличение органа в объеме. Какой вид нарушения кровообращения наблюдается при этом?

A.Рабочая гиперемия

B.Метаболическая артериальная гиперемия

C.*Венозная гиперемия

D.Реактивная гиперемия

E.Ишемия

21.У больного с тромбофлебитом нижних конечностей внезапно после нагрузки возникла одышка, резкая боль в груди, цианоз, набухание шейных вен. Тромбоэмболия каких из перечисленных сосудов наиболее вероятно вызвала такие нарушения?

A.Венечных сосудов

B.Воротной вены

C.Сосудов головного мозга

D.*Легочной артерии

E.Мезентериальных сосудов

22.У пациента с хронической сердечной недостаточностью выявлено увеличение вязкости крови, при капилляроскопии обнаружено повреждение стенок сосудов микроциркуляторного русла. Развитие какого из перечисленных нарушений возможно в данном случае?

A.*Венозной гиперемии

B.Артериальной гиперемии

C.Тромбоза

D.Сладж-феномена

E.Эмболии

23.В эксперименте у кролика был удален верхний шейный узел симпатического ствола. На стороне удаления наблюдаются покраснение и повышение температуры кожи головы. Какая форма нарушений периферического кровообращения развилась у кролика?

A.Стаз

B.*Нейропаралитическая артериальная гиперемия

C.Венозная гиперемия

D.Нейротоническая артериальная гиперемия

E.Метаболическая артериальная гиперемия

24.У больного 45 лет жалобы на кровотечение из десен при чистке зубов. Десна темнокрасного цвета, при надавливании гноетечение из десневых уголков. Обнаружен парадонтит. Какое местное нарушение кровообращения имеет место в ткани десен больного?

A.*Венозная гиперемия

B.Стаз

C.Ишемия

D.Тромбоз

E.Эмболия

25.Женщина 42-х лет с невралгией тройничного нерва жалуется на периодическое покраснение правой половины лица и шеи, ощущение прилива тепла и повышение кожной чувствительности. Какая артериальная гиперемия по механизму возникновения наблюдается в данном случае?

A.Реактивная

B.Рабочая

C.*Нейротоническая

D.Нейропаралитическая

E.Метаболическая

26.Женщина обратилась к врачу с жалобами на боли в ногах, которые появляются к вечеру, отечность стоп и голеней. Объективно: кожа на ногах синюшного цвета , холодная на ощупь. Какой тип нарушения периферического кровообращения имеется у данной больной?

A.*Венозная гиперемия

B.Тромбоз

C.Артериальная гиперемия

D.Стаз

E.Ишемия

27.Пациент 54-х лет после значительного психоэмоционального напряжения ощутил сильную боль за грудиной с иррадиацией в левую руку, левую половину шеи, страх смерти, он покрылся холодным потом. Прием нитроглицерина успокоил боль . Назовите расстройство местного кровообращения в сердце, которое наиболее вероятно развилось в данном случае:

A.Артериальная гиперемия

B.Венозная гиперемия

C.Эмболия

D.Тромбоз

E.*Ишемия

28.У ребёнка вследствие попадании горячей воды на руку возник ожог. Кожа на месте ожога ярко-красная. Какое нарушение местного ковообращения возникло на месте ожога?

A.Тромбоз

B.Венозная гиперемия

C.*Артериальная гиперемия

D.Эмболия

E.Стаз

29.У больного с переломом бедренной кости в области хирургической шейки появились признаки острой правожелудочковой недостаточности вследствие эмболии легочной артерии. Какой вид эмболии имеет место?

A.*Жировая

B.Тканевая

C.Газовая

D.Метастатическая

E.Воздушная

30.У водолаза, длительное время пребывающего на глубине 40 м, при декомпрессии развилась кесонная болезнь. Основным патогенетическим звеном стала эмболия:

A.Воздушная

B.Жировая

C.Тканевая

D.*Газовая

E.Пародоксальная

31.Студент на экзамене не смог правильно ответить на вопросы экзаменационного билета, что сопровождалось покраснением кожи лица, чувством жара и неуверенностью поведения. Какой вид артериальной гиперемии развился в данном случае?

A.Патологическая

B.Метаболическая

C.Постишемическая

D.*Нейротоническая

E.Нейропаралитическая

32.При воспалении глаза у больного отмечалось накопление мутной жидкости с высоким содержанием белка на дне передней камеры, получившее название - гипопион. Какой процесс лежит в основе указанных изменений?

A.*Нарушение микроциркуляции

B.Первичная альтерация

C.Пролиферация

D.Вторичная альтерация

33.У больного обнаружено нарушение периферического кровообращения, основой которого является ограничение прилива артериальной крови. При этом имеет место побледнение данного участка, снижения местной температуры. Как называется это явление?

A.*Ишемия

B.Артериальная гиперемия

C.Венозная гиперемия

D.Сладж

E.Лимфостаз

34. У больного 40-ка лет течение с ишемической болезнью сердца и заболеванием сосудов ног (облитерирующий эндартериит) при осмотре нижних конечностей обнаружена бледность и дистрофические изменения кожи, снижение местной температуры, нарушение чувствительности, боль. Какое нарушение периферического кровообращения имеется у больного?

A. *Обтурационная ишемия

B.Компрессионная ишемия

C.Ангиоспастическая ишемия

D.Венозная гиперемия

E.Артериальная гиперемия

35. После наложения жгута у исследуемого больного обнаружили точечные кровоизлияния. Из нарушениям функции каких клеток это связано?

A.*Тромбоцитов

B.Моноцитов

C.Эозинофилов

D.Нейтрофилов

E.Лимфоцитов

36. При смазывании скипидаром языка кролика он алеет, его кровенаполнение увеличивается. Артериальная гиперемия какого типа возникает в этом случае?

A.Нейротоническая

B.Нейропаралитическая

C.*Метаболическая

D.Реактивная

E.Рабочая

37.Человеку в глаз попал тополиный пух, который повлек раздражение и покраснение глаза. Какой вид расстройства местного кровообращения наблюдается?

A.*Артериальная гиперемия

B.Ишемия

C. Эмболия

.

D.Венозная гиперемия

E.Тромбоз

38.В состоянии испуга отмечается резкое побледнение лица, дрожь в конечностях. Какой вид ишемии наблюдается в данном случае?

A.*Ангиоспастическая

B.Компрессионная

C.Обтурационная (тромбообразование)

D.Метаболическая

E.Обтурационная (утолщение сосудистой стенки)

39.Участок кожи, который подвергался действию высокой температуры, покраснел. Назовите тип расстройства местного кровообращения в очаге острого воспаления, которое обусловливает такой его симптом, как “rubor”.

A.*Артериальная гиперемия

B.Венозная гиперемия

C.Ишемия

D.Стаз

E.Тромбоз

40.В эксперименте К. Бернар, раздражая chorda tympani (ветви n. facialis), наблюдал усиление секреции подчелюстной слюнной железы и развитие артериальной гиперемии. Какой по механизму развития является эта гиперемия?

A.*Нейротоническая

B.Нейропаралитическая

C.Метаболическая

D.Реактивная

E.Рабочая

41.При разгерметизации кабины самолета на высоте 19 км наступила мгновенная смерть пилотов. Какова ее причина?

A.*Закипание крови

B.Кровоизлияние в головной мозг

C.Газовая эмболия сосудов сердца

D.Кровотечение

E.Паралич дыхательного центра

42. Больному с закрытым переломом плечевой кости наложена гипсовая повязка. На следующий день появилась припухлость, синюшность и похолодание кисти травмированной руки. О каком расстройстве периферического кровообращения свидетельствуют эти признаки?

A.*Венозной гиперемии

B.Артериальной гиперемии

C.Ишемии

D.Тромбоза

E.Эмболии

43.После вынужденного быстрого поднятия водолаза из глубины на поверхность у него появились признаки кессонной болезни – боль в суставах, зуд кожи, мерцание в глазах, затемение сознания. Каким видом эмболии они были предопределены?

A.*Газовой

A.Воздушной

B.Жировой

C.Тканевой

D.Тромбоемболией

44.У больного с облитерирующим эндартериитом проведена ганглионарная симпатэктомия. Какой вид артериальной гиперемии возник в результате операции?

A.*Нейропаралитическая

B.Нейротоническая

C.Метаболическая

D.Рабочая

E.Реактивная

45.У больной 65-ти лет, страдающей тромбофлебитом глубоких вен голени, в поликлинике в очереди на прием к врачу, внезапно наступила смерть. На вскрытии труппа в общем стволе и бифуркации легочной артерии найдены свободно лежащие красные рыхлые массы с тусклой гофрированой поверхностью. Какой патологический процесс в легочной артерии нашел патологоанатом?

A.*Тромбоэмболию

B.Тромбоз

C.Тканевую эмболию

D.Эмболию инородными телами

E.Жировую эмболию

46.У людини в місці травми спостерігається почервоніння шкіри, пульсація дрібних артерій, підвищення місцевої температури, підвищення тургору тканини. Для якого

розладу місцевого кровообігу це характерно?

A.Ішемії

B.Венозної гіперемії

C.Тромбозу

D.Емболії

E.*Артеріальної гіперемії

47.У хворого із гострою лівошлуночковою недостатністю виник набряк легень. Яке порушення периферичного кровообігу в легенях стало причиною цього ускладнення?

A.Ішемія

B.Артеріальна гіперемія нейротонічного типу

C.Артеріальна гіперемія нейропаралітичного типу

D.Тромбоз легеневої артерії

E.*Венозна гіперемія

48.У хворого 54-х років, який скаржиться на біль, блідість та відчуття на похолодання нижніх кінцівок, лікар діагностував облітеруючий ендарартеріїт. Яке порушення периферичного кровообігу є головною причиною зазначених симптомів?

A.Венозна гіперемія

B.Нейропаралітична артеріальна гіперемія

C.Нейротонічна артеріальна гіперемія

D.*Обтураційна ішемія

E.Венозний стаз

49.У пацієнта зі скаргами на болі в правій нозі при огляді стопи виявлено: збліднення, зменшення об'єму, місцеве зниження температури. Яке порушення місцевого кровообігу спостерігається у хворого?

A.*Ішемія

B.Венозна гіперемія

C.Нейротонічна артеріальна гіперемія

D.Нейропаралітична артеріальна гіперемія

E.Метаболічна артеріальна гіперемія

50.У больного наблюдаются отечность и цианоз правой голени, расширение вен, местное снижение температуры. Какое нарушение периферического кровообращения отмечается у больного?

A.Ишемия

B.*Венозная гиперемия

C.Рабочая артериальная гиперемия

D.Метаболическая артериальная гиперемия

E.Эмболия

51.Для експериментального утворення тромбів в судинах поруч з веною брижі жаби кладуть кристалик кухонної солі. Що є основним механізмом, який запускає тромбоутворення у данному випадку?

A.Завихрення кровотоку.

B.Сповільнення кровотоку.

C.*Пошкодження ендотелію.

D.Зростання активності системи згортання крові.

E.Зниження активності системи протизгортання крові.

52.В експерименті у вені брижі жаби визвано утворення тромбу за допомогою

кристаллика кухонної солі. Які процеси відбуваються у першій фазі тромбоутворення?

A.Утворення тромбіну.

B.Утворення активного тромбопластину.

C.*Адгезія, агрегація, аглютинація тромбоцитів.

D.Утворення фібрину-мономеру.

E.Утворення фібріну-полімеру.

53.Одним из следствий нарушения кровообращения при воспалении является эксудация. В какую стадию сосудистых расстройств при воспалении она начинается?

A.Стаза

B.Венозной гиперемии

C.Спазма артериол

D.Престаза

E.*Артериальной гиперемии

54.У больного после физической нагрузки развился приступ стенокардии вследствие ишемии миокарда. Какая из формулировок наиболее точно отражает определение ишемии?

A.*Несоответствие между притоком крови к тканям и потребностью в ней

B.Уменьшение количества эритроцитов в крови

C.Расширение артериол

D.Дефицит кислорода в крови

E.Увеличение доставки кислорода в ткани

55.У хворого 45-ти років скарги на кровотечу з ясен при чистці зубів. Ясна темночервоного кольору, при натисканні гноєтеча з ясеневих закутків. Виявлений парадонтит. Яке місцеве порушення кровообігу має перевагу у тканині ясен хворого?

A.*Венозна гіперемія.

B.Ішемія.

C.Стаз.

D.Тромбоз.

E.Емболія.

56.У хворого з пораненням кисті почав утворюватися набряк. У якій стадії порушення місцевого кровообігу це відбувається?

A.*Артеріальна гіперемія

B.Стаз

C.Венозна гіперемія

D.Спазм артеріол

E.Престаз

57.У больного с варикозным расширением вен, при осмотре нижних конечностей отмечается: цианоз, пастозность, снижение температуры кожи, единичные петехии. Какое расстройство гемодинамики имеется у больного?

A.*Венозная гиперемия

B.Компрессионная ишемия

C.Обтурационная ишемия

D.Тромбэмболия

E.Артериальная гиперемия

58.После механической травмы больному наложили жгут на руку, чтобы остановить кровотечение. Ниже жгута рука побледнела, появилось чувство онемения. Это

состояние является следствием:

A.*Компрессионной ишемии

B.Венозного застоя

C.Обтурационной ишемии

D.Ангиоспастической ишемии

E.Тромбоза

59.Хворий 25-ти років скаржиться на появу і посилення болю в м’язах ніг під час ходьби, через що він змушений часто зупинятись. Об’єктивно: шкіра на ногах бліда, волосяний покрив відсутній, нігті на пальцях стоп – з трофічними змінами. Пульсація на артеріях стоп відсутня. Найвірогідніше причиною цих змін є:

A.* Ішемія.

B.Венозна гіперемія.

C.Артеріальна гіперемія.

D.-

E.Емболія.

60.У хворого після лікування карієсу в ділянці ясен навколо хворого зуба виникла гіперемія, набряк, біль. Яке порушення місцевого кровообігу виникло в даному випадку?

A.*Венозна гіперемія

B.Тромбоз

C.Ішемія

D.Стаз

E.Престаз

61.Хворому 28-ми років, з переломом плечової кістки накладена гіпсова пов’язка. Наступного дня з’явилися припухлість, синюшність і похолодання кисті травмованої руки. Про який розлад периферичного кровообігу свідчать ці ознаки?

A.*Венозна гіперемія

B.Артеріальна гіперемія

C.Ішемія

D.Тромбоз

E.Емболія

62.При синдроме реперфузии активируются процессы свободнорадикального окисления, что приводит к повреждению клеточных мембран и нарушению специфических функций клеток. Эти изменения связаны с избыточным накоплением в цитоплазме ионов:

А. Калия

B.Магния

C.Хлора

D.Натрия

E.*Кальция

63.У женщины 25-ти лет на 8-м месяце беременности появились признаки расширения вен нижних конечностей, отеки стоп. Какой вид расстройств периферического кровообращения наблюдается у больной?

A.Эмболия

B.Артериальная гиперемия нейропаралитического типа

C.*Венозная гиперемия

D.Артериальная гиперемия нейротонического типа

E.Ишемия

64.В аварийной ситуации аквалангист быстро поднялся с глубины на поверхность. У него отмечается потеря сознания, нарушение дыхания и сердечной деятельности в результате развития кессонной болезни. Какое осложнение может развиться у аквалангиста?

A.Воздушная эмболия

B.Тромбоэмболия

C.*Газовая эмболия

D.Клеточная эмболия

E.Жировая эмболия

65.При моделюванні запалення на брижі жаби під мікроскопом спостерігали розширення артеріальних судин, прискорення кровотоку, осьовий ток крові. Який вид артеріальної гіперемії виник при цьому?

A.*Метаболічна

B.Постішемічна

C.Вакатна

D.Реактивна

E.Робоча

66.У хворого 29-ти років з діагнозом: багатооскольчастий перелом правого стегна на 3 добу від одержання травми з’явилися скарги на біль у грудній порожнині зправа, утруднене дихання. Через добу на фоні прогресуючої серцево-дихальної недостатності настала смерть. При гістологічному дослідженні у кровоносних судинах легень та головного мозку виявлені суданофільні краплини оранжевого кольору, які повністю перекривали просвіти судин мікроциркуляторного руслу. З яким ускладненням пов’язана смерть хворого?

A.*Жировою емболією

B.Газовою емболією

C.Медикаментозною емболією

D.Мікробною емболією

E.Тромбоемболією

Соседние файлы в папке Патофизиология